Heart Murmurs Flashcards

1
Q

–occasionally experiences a “forceful” heartbeat, but denies chest pain, dizziness or syncope.

– patients often have a widened pulse pressure (the difference between the systolic and diastolic pressures) (ex. 140/40)

–Palpation of the precordium shows laterally displaced point of maximal impulse.

–The lungs do not have any adventitious sounds. There is no peripheral edema.

A

Aortic regurgitation

How well did you know this?
1
Not at all
2
3
4
5
Perfectly
2
Q

a low-pitched diastolic rumble with presystolic accentuation, best heard at the apex with the patient in the left lateral decubitus position.

A
  • Mitral stenosis
  • rheumatic heart disease.
  • Exertional dyspnea, orthopnea, PND
  • Mitral stenosis results in elevated left atrial and pulmonary venous pressure leading to pulmonary congestion.
  • May lead to afib if long standing
  • Sx increase with exercise and during pregnancy
How well did you know this?
1
Not at all
2
3
4
5
Perfectly
3
Q

Deficiency in ______________ leads to peripheral neuropathy, seborrheic dermatitis, and glossitis. Depression and confusion has been associated with this deficiency. Consider __________deficiency in seizures refractory to anti-convulsant medication in both adults and infants.

A

Vitamin

Pyridoxine

How well did you know this?
1
Not at all
2
3
4
5
Perfectly
4
Q

Aortic stenosis+ valsalva maneuver

A

Decreased intensity

The Valsalva maneuver interferes with venous return to the right side of the heart, and consequently reduces right ventricular preload. This also results then, in a decrease in left ventricular end-diastolic volume. The reduced pressure gradient between the left ventricle and aorta causes blood flow and velocity to also decrease, resulting in less turbulence. The outcome then is a decrease in the intensity of the murmur.

How well did you know this?
1
Not at all
2
3
4
5
Perfectly
5
Q

early diastolic murmur heard best over the (right 2nd intercostal space).

A

Aortic regurgitation

How well did you know this?
1
Not at all
2
3
4
5
Perfectly
6
Q

Tetralogy of Fallot

A

One acronym that can be useful in remembering the Tetralogy is “PROVe”

Pulmonary stenosis

Right ventricular hypertrophy

Overriding aorta

Ventricular septal defect

How well did you know this?
1
Not at all
2
3
4
5
Perfectly
7
Q

A 45-year-old woman comes to the clinic with joint pain in her knees and wrists, and fever for the past week. On physical examination you notice the presence of erythematous annular lesions on the trunk and extremities. Patient states that she was recently seen in the urgent care for a sore throat and was treated with supportive therapy. Which of the following is the most likely diagnosis?

A

Rheumatic fever

This is a patient presenting with rheumatic fever. It usually occurs 2-4 weeks after group A streptococcus pharyngitis. The major criteria for rheumatic fever include: polyarthritis, carditis, erythema marginatum, chorea, subcutaneous nodules. Minor criteria include: fever, polyarthralgias, reversible prolongation of PR interval, elevated ESR and CRP. The diagnosis require 2 major criteria, or 1 major and 2 minor. Joint pain is typically the earliest symptom.

How well did you know this?
1
Not at all
2
3
4
5
Perfectly
8
Q

“apple core” lesion in a barium enema study

A

Colon Cancer

Patients with colorectal cancer may be asymptomatic, or complain of abdominal pain, rectal bleeding, or a change in bowel habits. Barium contrast x-rays will show filling defects due to colonic masses seen in colon cancer. Cancers that produce a ‘apple-core’ appearance on barium enema are circumferential and cause narrowing of the lumen

How well did you know this?
1
Not at all
2
3
4
5
Perfectly
9
Q

A 2-year-old boy is brought to the emergency department by his parents because they think that he swallowed a quarter. Chest x-ray study is shown. In which of the following locations is the pictured foreign body most likely located?

A

Esophagus

Coins aspirated into the trachea or swallowed down the esophagus are ‘classically’ differentiated based upon their orientation on the anteroposterior chest x-ray study. Coins aspirated through the vocal cords are traditionally described as having an “end-on” vertical orientation. Those in the esophagus are seen as radiopaque disks. Coins account for ~70% of pediatric ingested foreign bodies, and will typically become ‘stuck’ at the level of the cricopharyngeus muscle.

How well did you know this?
1
Not at all
2
3
4
5
Perfectly
10
Q

RUQ abdominal pain, fever, jaundice

A

Charcot’s triad is the classical presentation of a patient with acute cholangitis: obstruction and infection of the bile ducts

How well did you know this?
1
Not at all
2
3
4
5
Perfectly
11
Q

diverticulitis tx

A

Metronidazole and ciprofloxacin

Diverticulitis is characterized by inflammation of colonic diverticula. Patients most commonly have left sided abdominal pain, diarrhea, fever, and nausea. Metronidazole and ciprofloxacin are the current recommended antibiotics for a patient with suspected diverticulitis, to target E. coli and anaerobes. Antibiotics should be given for 7-10 days. Complications of diverticulitis include abscess formation, obstruction, and colonic perforation.

How well did you know this?
1
Not at all
2
3
4
5
Perfectly
12
Q

Elicitation of pain when pushing on the tragus

A

external otitis.

How well did you know this?
1
Not at all
2
3
4
5
Perfectly
13
Q

In which of the following anatomical locations are peptic ulcers most likely to occur?

A

A. Lesser curvature of the stomach and the duodenal bulb

This is the correct answer and historically most patients with peptic ulcer disease have ulcers in one if not both of these locations.

How well did you know this?
1
Not at all
2
3
4
5
Perfectly
14
Q

Wernicke-Korsakoff syndrome

A

Thiamine (vitamin B-1) deficiency can cause beriberi, and Wernicke-Korsakoff syndrome.

How well did you know this?
1
Not at all
2
3
4
5
Perfectly
15
Q

the most common cause of intestinal obstruction in children between 3 months and 3 years old. It is caused by one segment of the intestines folding into another segement and causing a telescoping effect.

A

Diagnostic and therapeutic: Air enema

Intussusception is the most common cause of intestinal obstruction in children between 3 months and 3 years old. It is caused by one segment of the intestines folding into another segement and causing a telescoping effect.

How well did you know this?
1
Not at all
2
3
4
5
Perfectly
16
Q

A 72 year old female presents to the emergency room with complaints of chest pain, dyspnea, palpitations, and a near-syncopal episode. Vital signs are BP 86/55, HR 115, RR 18, O2Sat 99% room air. Physical exam reveals altered mental status, weak peripheral pulses, jugular venous distention and 2+ pitting edema of bilateral lower extremities to the knees. Auscultation of bilateral lower lung bases reveals wet crackles. Which of the following is the drug of choice in the treatment of this patient?

A

Dobutamine

This is the classic presentation of a patient with cardiogenic shock. Dobutamine is the preferred drung in this patient because she is tachycardic and dobutamine does not increase myocardial oxygen demand like dopamine does. Volume replacement is also essential in treating all cases of shock.

How well did you know this?
1
Not at all
2
3
4
5
Perfectly
17
Q

Active hip flexion of the thigh against resistance

A

psoas sign

How well did you know this?
1
Not at all
2
3
4
5
Perfectly
18
Q

radiates to carotids

A

aortic stenosis

How well did you know this?
1
Not at all
2
3
4
5
Perfectly
19
Q

prinzmetal angina tx

A

CCBs

Calcium channel blockers are used to treat and prevent it

CCBs cause vasodilation of the coronary arteries and have been shown to both treat and prevent coronary artery spasm.

How well did you know this?
1
Not at all
2
3
4
5
Perfectly
20
Q

Physical examination shows distention of the upper abdomen, and a 1.5-2 cm diameter palpable firm, nontender moblie mass in the right upper quadrant at the lateral edge of the rectus abdominus muscle.

A

Pyloric stenosis

Ultrasound

Ultrasonography is the imaging modality of choice when evaluating a child for infantile hypertrophic pyloric stenosis. In the hands of a qualified sonographer, it is both highly sensitive (90-99%) and specific (97-100%).

How well did you know this?
1
Not at all
2
3
4
5
Perfectly
21
Q

Pellagra

A

Niacin (vitamin B-3)

How well did you know this?
1
Not at all
2
3
4
5
Perfectly
22
Q

Diastolic rumble heard best over the left 4th intercostal space

A

tricuspid stenosis.

How well did you know this?
1
Not at all
2
3
4
5
Perfectly
23
Q

Central retinal artery occlusion (CRAO) is generally caused by

A

CRAO by embolism.

How well did you know this?
1
Not at all
2
3
4
5
Perfectly
24
Q

Abdominal radiographs show distended loops of colon with prominent haustral markings

A

LBO

This patient is presenting with symptoms suggestive of colonic obstruction

The most common cause of LBO is adenocarcinoma (60%), followed by diverticular disease (20%), and volvulus (5%).

How well did you know this?
1
Not at all
2
3
4
5
Perfectly
25
Q

Heterophile antibody testing

A

Mononucleosis

How well did you know this?
1
Not at all
2
3
4
5
Perfectly
26
Q

A 79 year old male presents to the emergency department for evaluation of blood in his stool. His wife mentions that over the last six months his appetite has decreased and he has lost around 20 pounds. Physical exam reveals enlarged lymph nodes periumbilically as well as in the supraclavicular notch. Which of the following is the most likely diagnosis?

A

Metastatic gastric cancer

Virchow’s node (left supraclavicular adenopathy) and Sister Mary Joseph’s node (periumbilical adenopathy) are suggestive of cancer metastasizing via the lymphatic system. Given the patient history of blood in the stool with constitutional symptoms, gastrointestinal etiology is most likely in this patient.

How well did you know this?
1
Not at all
2
3
4
5
Perfectly
27
Q

complete ipsilateral vision loss

A

Optic nerve

An optic nerve lesion would cause a complete ipsilateral vision loss (a lesion of the right optic nerve would causes complete right eye blindness).

How well did you know this?
1
Not at all
2
3
4
5
Perfectly
28
Q

leads I, aVL, V5 and V6.

A

lateral

How well did you know this?
1
Not at all
2
3
4
5
Perfectly
29
Q

radiating to axilla

A

mitral regurgitation

How well did you know this?
1
Not at all
2
3
4
5
Perfectly
30
Q

Oral hairy leukoplakia is associated with _________

A

Epstein-Barr virus and HIV infection.

How well did you know this?
1
Not at all
2
3
4
5
Perfectly
31
Q

Placement of a patient in the left lateral decubitus position and then passively extending the patient’s right leg at the hip, with knee extended, best represents which of the following physical exam techniques?

A

Psoas sign

This correctly describes psoas sign. If right lower quadrant pain is elicited when performing this maneuver, it raises suspicion of acute appendicitis.

How well did you know this?
1
Not at all
2
3
4
5
Perfectly
32
Q

Carcinogenic embryonic antigen (CEA)

A

This tumor marker is seen in colon cancer.

How well did you know this?
1
Not at all
2
3
4
5
Perfectly
33
Q

Patients will have a red, painful eye and fixed mid-dilated pupil. Nausea and vomiting can also occur.

A

acute, angle-closure glaucoma?

A. Painful, red eye

Acute angle-closure glaucoma is the rapid narrowing of the anterior chamber angle, thereby preventing drainage of aqueous fluid through the trabecular meshwork. This is an ophthalmic emergency. Patients will have a red, painful eye and fixed mid-dilated pupil. Nausea and vomiting can also occur. Emergent evaluation by an ophthalmologist is critical.

In acute angle-closure glaucoma, onset of symptoms more commonly occur in the evening, or when in a dark environment. Low light causes dilation of the pupil, which can further block the anterior chamber angle.

How well did you know this?
1
Not at all
2
3
4
5
Perfectly
34
Q

classic triad of chest pain, dizziness (or syncope), and heart failure.

A

Aortic valve stenosis

Patients with aortic stenosis (AS) have a classic triad of chest pain, dizziness (or syncope), and heart failure. AS produces a systolic murmur, delayed carotid pulses, and a S4 gallop due to a forceful atrial contraction into a hypertrophied left ventricle.

How well did you know this?
1
Not at all
2
3
4
5
Perfectly
35
Q

used in an overdose of benzodiazepines.

A

Flumazenil

How well did you know this?
1
Not at all
2
3
4
5
Perfectly
36
Q

An abdominal CT is the diagnostic study of choice, and will show a “whirl pattern” of the dilated proximal colon.

A

Volvulus

Volvulus accounts for around 5% of cases of LBO, with sigmoid volulus being the most common. It occurs more often in the elderly and in patients with chronic constipation. Patients with have abdominal pain, distension, constipation. Vomiting is less common. An abdominal CT is the diagnostic study of choice, and will show a “whirl pattern” of the dilated proximal colon. Sigmoidosocopy can be done to detorse the volvulus.

How well did you know this?
1
Not at all
2
3
4
5
Perfectly
37
Q

A 32-year-old man with diabetes comes to the emergency department with two days of intense, deep ear pain and a purulent drainage from the left ear. Physical examination reveals a erythematous and edematous canal which contains some debris and granulation tissue. The tympanic membrane is barely visible but appears intact. The surrounding external ear is also indurated and erythematous. The patient has a fever of 102.1F. Which of the following is the next best step in the care of this patient?

A

TX=

Admit the patient to the hospital for intravenous antibiotics

In diabetic or immunocompromised patients, the risk of malignant otitis externa (osteomyelitis of the skull base) is high, and treatment should begin immediately with IV anti-pseudomonal antibiotics. Ciprofloxacin is the first-line antibiotic. A CT may also be done to evaluate for bone erosion.

How well did you know this?
1
Not at all
2
3
4
5
Perfectly
38
Q

Which of the following electrolyte disturbances is most likely to occur with use of angiotensin converting enzyme inhibitors

A

Hyperkalemia

How well did you know this?
1
Not at all
2
3
4
5
Perfectly
39
Q

Cardiac tamponade is a condition that may be life threatening. Which of the following mechanisms correctly describes how stroke volume is impeded in this condition?

A

Cardiac chambers are compressed, decreasing preload

This is the correct mechanism by which the effusion in tamponade decreases stroke volume.

How well did you know this?
1
Not at all
2
3
4
5
Perfectly
40
Q

Retinal hemorrhages on fundoscopic exam should inspire what other test in babies?

A

Skeletal survey

Retinal hemorrhages on fundoscopic exam should always increase suspicion for Abusive Head Trauma. “Shaken Baby Syndrome” has characteristic clinical features of retinal hemorrhages (often bilateral, multilayered, and extensive), subdural hematoma, and/or occult fractures (particularly of the ribs and long bone metaphyses). If this is suspected a skeletal survey should be ordered to assess for further injury that may have been inflicted during the abuse.

How well did you know this?
1
Not at all
2
3
4
5
Perfectly
41
Q

Chronic aortic regurgitation leads to __________, which can be manifested by a laterally displaced point of maximal impulse and anbormal EKG findings.

A

left ventricular hypertrophy

How well did you know this?
1
Not at all
2
3
4
5
Perfectly
42
Q

______________ inhibits reabsorption of sodium and chloride ions from the distal convoluted tubules by blocking the sodium-chloride symporter.

A

Thiazide diuretics (e.g. hydrochlorothiazide) inhibits reabsorption of sodium and chloride ions from the distal convoluted tubules by blocking the sodium-chloride symporter.

How well did you know this?
1
Not at all
2
3
4
5
Perfectly
43
Q

PR intervals are constant and then a QRS complex is “dropped”

A

Mobitz Type II is characterized by a costant PR interval (may be normal or long in duration) and then a QRS complex is occasionally dropped.

How well did you know this?
1
Not at all
2
3
4
5
Perfectly
44
Q

Low frequency diastolic murmur heard best at the apex

A

mitral stenosis.

How well did you know this?
1
Not at all
2
3
4
5
Perfectly
45
Q

Which of the following best describes the pressure against which the heart must work to eject blood during systole?

A

afterload

How well did you know this?
1
Not at all
2
3
4
5
Perfectly
46
Q

Passive hip flexion and internal rotation

A

obturator sign

How well did you know this?
1
Not at all
2
3
4
5
Perfectly
47
Q

most diverticula occur in the ______

A

sigmoid colon.

How well did you know this?
1
Not at all
2
3
4
5
Perfectly
48
Q

What is the most common cause of viral conjunctivitis?

A

Adenovirus

Adenoviral conjunctivitis is the most common viral conjunctivitis.

How well did you know this?
1
Not at all
2
3
4
5
Perfectly
49
Q

Neural tube defects

A

Folate (vitamin B-9)

How well did you know this?
1
Not at all
2
3
4
5
Perfectly
50
Q

A 27-year-old man is seen in the Emergency Department for right ear pain one hour after being assaulted. He was struck once by an assailant’s fist directly over his right ear. He does not have any other injuries or neck pain, and he did not have any loss of consciousness. Physical examination of the anterior auricle shows a tense, buldging 1 cm mass underneath the skin overlying the ear cartilage, along with swelling, deformity, and loss of auricular landmarks. The remainder of the examination shows no abnormalities. Which of the following is the most appropriate definitive management of this patient’s conditon?

A

Incision and drainage

Auricular hematoma occurs when a blunt shearing force is sustained to the ear (e.g. boxing). These forces separate the underlying perichondrium, which is highly vascularized, from the cartilage itself. The result is formation of a hematoma between this space which disrupts blood flow to the cartilage. If left untreated, auricular hematoma may subsequently lead to cartilage necrosis with formation of a ‘cauliflower’ ear. Initial definitive treatment of choice for this condition is incision and drainage followed by application of a compression dressing to prevent reaccumulation of blood.

How well did you know this?
1
Not at all
2
3
4
5
Perfectly
51
Q

Venous stasis, hypercoagulability, vascular injury

A

Virchow’s triad, which outlines the pathogenesis of venous thromboembolism (VTE) formation

How well did you know this?
1
Not at all
2
3
4
5
Perfectly
52
Q

______________is the most common cause of brisk hematochezia, accounting for 30-50% of cases of rectal bleeding.

A

Colonic diverticular bleeding

How well did you know this?
1
Not at all
2
3
4
5
Perfectly
53
Q

Distended neck veins, distant heart sounds, hypotension

A

Beck’s triad, which describes the classical findings of cardiac tamponade.

How well did you know this?
1
Not at all
2
3
4
5
Perfectly
54
Q

Peritonsillar abscess: The uvula may be displaced towards the ___________side.

A

unaffected

How well did you know this?
1
Not at all
2
3
4
5
Perfectly
55
Q

Kawasaki disease treatment

A

IVIG and aspirin are the standard treatment for Kawasaki disease.

How well did you know this?
1
Not at all
2
3
4
5
Perfectly
56
Q

undercooked ground beef and unpasteurized juice and milk. Symptoms include severe abdominal cramping and bloody diarrhea. Fever is a less common symptom.

A

Escherichia coli

How well did you know this?
1
Not at all
2
3
4
5
Perfectly
57
Q

H. Pylori tx

A

Omeprazole, amoxicillin, clarithromycin

Metronidazole is only used in patients who are penicillin allergic.

A positive urea breath test in the appropriate clinical setting is indicative of an active Helicobacter pylori infection. Other noninvasive diagnostic tests include stool antigen test and serology tests. Once diagnosed, H. pylori infections are classically treated with the triple therapy consisting of proton-pump inhibitor, amoxicillin, and clarithromycin for 14 days.

How well did you know this?
1
Not at all
2
3
4
5
Perfectly
58
Q

diastolic opening snap

A

mitral stenosis

How well did you know this?
1
Not at all
2
3
4
5
Perfectly
59
Q

Diverticulosis most often occurs in which portion of the large intestine?

A

Sigmoid Colon

Diverticula can occur anywhere along the colon but are significantly more common in the sigmoid colon.

How well did you know this?
1
Not at all
2
3
4
5
Perfectly
60
Q

Which of the following electrocardiogram abnormalities is found in Cor Pulmonale?

A

Right ventricular hypertrophy

Right ventricular hypertrophy is the most common ECG finding seen in cor pulmonale. Cor pulmonale is the reduced function of the right ventricle secondary to pulmonary disease and pulmonary hypertension. Chronic cor pulmonale is commonly caused by COPD, and acute cor pulmonale can occur with PE and ARDS.

How well did you know this?
1
Not at all
2
3
4
5
Perfectly
61
Q

most common primary cancer of the eye in adults. It occurs most often in lightly pigmented individuals, and the median age of occurrence is 55 years.

A

Ocular melanoma

Ocular melanoma is the most common primary cancer of the eye in adults. It occurs most often in lightly pigmented individuals, and the median age of occurrence is 55 years.

How well did you know this?
1
Not at all
2
3
4
5
Perfectly
62
Q

Which of the following is the most common primary site of gastrointestinal malignancy in the United States?

A

colorectal

How well did you know this?
1
Not at all
2
3
4
5
Perfectly
63
Q

present with pain, a foreign body sensation, and often can relay a history of trauma or doing an activity where something may have scratched the cornea.

A

Corneal abrasion

Patients with a corneal abrasion present with pain, a foreign body sensation, and often can relay a history of trauma or doing an activity where something may have scratched the cornea.

How well did you know this?
1
Not at all
2
3
4
5
Perfectly
64
Q

early diastolic, decrescendo murmur beginning with the pulmonary component of the second sound, and is heard best along the upper left sternal border.

A

pulmonary valve regurgitation (usually due to pulmonary hypertension)

How well did you know this?
1
Not at all
2
3
4
5
Perfectly
65
Q

Diastolic rumble heard best in the left 4th intercostal space

A

tricuspid stenosis.

How well did you know this?
1
Not at all
2
3
4
5
Perfectly
66
Q

Primary sclerosing cholangitis is strongly associated with what?

A

Primary sclerosing cholangitis

Primary sclerosing cholangitis (PSC) is an autoimmune disease characterized by the destruction of both the intra- and extrahepatic biliary ducts that may eventually lead to liver failure. It is strongly associated with ulcerative colitis.

How well did you know this?
1
Not at all
2
3
4
5
Perfectly
67
Q

Boot shaped contour of the heart

A

Tetralogy of Fallot is characterized by four congenital abnormalities: VSD, pulmonary artery stenosis, an overriding aorta, and right ventricular hypertrophy. Tetralogy of Fallot is characterized by “tet spells”, in which the infant becomes cyanotic when agitated, due to obstruction of the right ventricular outflow tract. A boot shaped heart on chest radiograph is classically associated with Tetralogy of Fallot.

How well did you know this?
1
Not at all
2
3
4
5
Perfectly
68
Q

serious complication that can occur 3-7 days following an MI. Patients present with evidence of heart failure including a new murmur, hypotension, and acute pulmonary edema.

During physical examination you auscultate a grade 2/6 mid-systolic murmur which was not documented on his last admission. You also auscultate diffuse wet crackles in bilateral lungs. The remainder of this physical examination reveals no abnormalities.

A

Papillary muscle rupture

During physical examination you auscultate a grade 2/6 mid-systolic murmur which was not documented on his last admission. You also auscultate diffuse wet crackles in bilateral lungs. The remainder of this physical examination reveals no abnormalities.

How well did you know this?
1
Not at all
2
3
4
5
Perfectly
69
Q

Which embryological remnant may remain open after birth, allowing a connection between the pulmonary artery and the aorta?

A

Ductus arteriosus

This is the embryologic remnant which may persist in patent ductus arteriosus (PDA). In utero, it connects the pulmonary artery to the aorta and diverts blood away from the nonfunctioning lungs. When it closes it becomes the ligamentum arteriosus.

How well did you know this?
1
Not at all
2
3
4
5
Perfectly
70
Q

Early diastolic murmur heard best over the right 2nd intercostal space

A

aortic regurgitation.

How well did you know this?
1
Not at all
2
3
4
5
Perfectly
71
Q

dilation of the proximal esophagus, and a distal narrowing with a bird’s beak appearance.

A

Achalasia

Esophageal achalasia is a motility disorder involving the smooth muscle layer of the esophagus, and the lower esophageal sphincter (LES). It is characterized by incomplete LES relaxation, increased LES tone, and lack of peristalsis of the esophagus (in the absence of other causes such as cancer or fibrosis). Achalasia is characterized by dysphagia, regurgitation, and sometimes chest pain. The dysphagia tends to progressively worsen over time. Diagnosis is confirmed with esophageal manometry, and barium swallow radiographic studies. Barium swallow study shows loss of normal peristaltic movement, and tapering of the lower esophageal sphincter with narrowing of the gastro-esophageal junction, producing a “bird’s beak” or “rat’s tail” appearance.

How well did you know this?
1
Not at all
2
3
4
5
Perfectly
72
Q

Which of the following is the most common complication of untreated atrial fibrillation?

A

Cerebral vascular accident

Because of the uncoordinated contracting of the atria, there is a propensity for the blood to pool and form clots. CVA or stroke is the most common complication associated with atrial fibrillation. For this reason, anticoagulation is an important consideration in treatment.

How well did you know this?
1
Not at all
2
3
4
5
Perfectly
73
Q

___________are usually only seen in patients with Zollinger–Ellison syndrome (ZES) which is an over production of acid. ZES accounts for a relatively small number of patients with PUD.

A

Jejunal ulcers

How well did you know this?
1
Not at all
2
3
4
5
Perfectly
74
Q

Shigella tx

A

Ciprofloxacin

Bactrim and fluoroquinolones are indicated in therapy of Shigella and Cholera.

How well did you know this?
1
Not at all
2
3
4
5
Perfectly
75
Q

Screening for strabismus in the pediatric population is performed in order to prevent what condition?

A

Amblyopia

Amblyopia is a condition in which visual acuity in one or both eyes is permanently compromised because of abnormal visual experiences during development of normal eyesight. This can be caused by strabismus (misalignment of the eyes). If detected, strabismus can be treated with refractive error correction, occlusion therapy, visual training exercises, and surgery.

How well did you know this?
1
Not at all
2
3
4
5
Perfectly
76
Q

Which of the following anti-hypertensive medications is contraindicated in patients with severe renal artery stenosis?

A

A. Enalapril

ACE Inhibitors are contraindicated in severe renal artery stenosis because they remove vasoconstriction on the efferent arteriole which maintains glomerular capillary pressure. Once this is removed the GFR decreases rapidly and the patient may enter renal failure.

How well did you know this?
1
Not at all
2
3
4
5
Perfectly
77
Q

Leads V1-V4

A

anterior leads

How well did you know this?
1
Not at all
2
3
4
5
Perfectly
78
Q

delayed carotid pulses

A

Aortic valve stenosis

Patients with aortic stenosis (AS) have a classic triad of chest pain, dizziness (or syncope), and heart failure. AS produces a systolic murmur, delayed carotid pulses, and a S4 gallop due to a forceful atrial contraction into a hypertrophied left ventricle.

How well did you know this?
1
Not at all
2
3
4
5
Perfectly
79
Q

The typical presentation is general overall feeling of being ill, inflammation to affected eye, decreased vision, and pain on movement of affected eye.

A

Orbital cellulitis

Orbital cellulitis is a dangerous infection that can result in lasting sequela. The typical presentation is general overall feeling of being ill, inflammation to affected eye, decreased vision, and pain on movement of affected eye.

How well did you know this?
1
Not at all
2
3
4
5
Perfectly
80
Q

Corrigan (“water hammer”) pulse

and a bobbing motion of the head with each heartbeat

A

aortic regurgitation

How well did you know this?
1
Not at all
2
3
4
5
Perfectly
81
Q

musty and mousy odor of the skin and urine.

A

Phenylketonuria

Phenylketonuria (PKU) is the most common inborn error of amino acid metabolism. It is caused by a deficiency of the enzyme phenylalanine hydroxylase which impairs the body’s ability to metabolize the essential amino acid, phenylalanine. This leads to accumulation of phenylalanine in body fluids. Elevated phenylalanine levels negatively impact cognitive function, and individuals with classic phenylketonuria almost always have intellectual disability unless levels are controlled through dietary or pharmacologic treatment. Physical examination findings in PKU include: fair skin and hair resulting from impairment of melanin synthesis, eczema, intellectual disability, ‘musty’ or ‘mousy’ odor, and seizures.

How well did you know this?
1
Not at all
2
3
4
5
Perfectly
82
Q

What increases mitral regurgitation?

A
  • Mitral regurgitation
  • pansystolic (holosystolic), blowing in quality, best heard at the apex, and often radiating to the left axilla
  • The murmur can increase in intensity with handgrip or squatting
  • Papillary muscle rupture/endocarditis
How well did you know this?
1
Not at all
2
3
4
5
Perfectly
83
Q

Plain abdominal radiographs will show colonic distention, usually of the transverse or right side of the colon.

A

Toxic Megacolon

Toxic megacolon is nonobstructive dilatation of the colon, with associated toxicity. It is a complication of inflammatory bowel disease and C. difficile infection. Plain abdominal radiographs will show colonic distention, usually of the transverse or right side of the colon.

(ulcerative colitis)

How well did you know this?
1
Not at all
2
3
4
5
Perfectly
84
Q

PR interval progressively increases until a QRS complex is “dropped”

A

Mobitz type I, also known as Wenckebach, is characterized as progressive PR interval prolongation until a QRS complex is dropped.

How well did you know this?
1
Not at all
2
3
4
5
Perfectly
85
Q

rheumatic fever

A

Mitral valve stenosis

The most common cause of mitral stenosis is rheumatic fever. Symptoms usually manifest in the third or fourth decade of life. Extreme exertion, excitement, or fever can provoke signs and symptoms including orthopnea and edema. Atrial dysrhythmias are common. The classic physical examination finding is a diastolic “rumbling” murmur with an opening snap.

How well did you know this?
1
Not at all
2
3
4
5
Perfectly
86
Q

Which of the following conditions is associated with recent infection with Campylobacter jejuni?

A

Guillain-Barré syndrome

Guillain-Barré syndrome (GBS) can be described as a collection of clinical syndromes that manifests as an acute inflammatory polyradiculoneuropathy with ascending weakness, and diminished reflexes. It is an autoimmune disease that destroys the myelin sheath, or even axons themselves. In two forms of GBS, axons are attacked by antibodies against Campylobacter jejuni. These antibodies react with proteins of the peripheral nerves. GBS most commonly occurs a few days or weeks after a respiratory or gastrointestinal infection.

How well did you know this?
1
Not at all
2
3
4
5
Perfectly
87
Q

acquired dilated cardiomyopathy, in which the ventricles dilate, resulting in impaired contraction and reduced systolic function.

A

Alcoholic cardiomyopathy

How well did you know this?
1
Not at all
2
3
4
5
Perfectly
88
Q

abdominal pain, jaundice, fever, hypotension, and mental status changes.

A

Reynold’s pentad describes severe cases of acute cholangitis, with symptoms of abdominal pain, jaundice, fever, hypotension, and mental status changes.

How well did you know this?
1
Not at all
2
3
4
5
Perfectly
89
Q

Which type of murmur is associated with papillary muscle rupture/endocarditis?

A
  • Mitral regurgitation
  • pansystolic (holosystolic), blowing in quality, best heard at the apex, and often radiating to the left axilla
  • The murmur can increase in intensity with handgrip or squatting
  • Papillary muscle rupture/endocarditis
How well did you know this?
1
Not at all
2
3
4
5
Perfectly
90
Q

most common organism responsible for cases of traveler’s diarrhea.

A

Escherichia coli

How well did you know this?
1
Not at all
2
3
4
5
Perfectly
91
Q

Which of the following is the most common clinical presentation of esophageal strictures?

A

Dysphagia

Progressive solid food dysphagia is the most common clinical presentation.

How well did you know this?
1
Not at all
2
3
4
5
Perfectly
92
Q

Plasma concentration of which of the following substances is most likely to be elevated following hepatocellular damage severe enough to cause hepatic encephalopathy?

A

Ammonia, tx=Lactulose

Hepatic encephalopathy is the occurrence of confusion, altered level of consciousness, and coma that results from liver failure. It is caused by accumulation of toxic substances in the bloodstream that are normally removed by the liver. The most important waste product is ammonia. Ammonia is absorbed and metabolized by brain astrocytes. Although ammonia plays a central role in hepatic encephalopathy, it is important to also note that ammonia levels do not always correlate with the severity of encephalopathy.

How well did you know this?
1
Not at all
2
3
4
5
Perfectly
93
Q

A 37-year-old man presents to the clinic with fever and a red rash which appears in ring formation covering his trunk and portions of his lower extremities. He also complains of bumps under the skin of his arms and legs that you recognize as subcutaneous nodules. Which of following heart murmurs is most commonly noted as a late sequelae of this disease?

A

Low-pitched diastolic rumble, possibly with opening snap

The mitral valve is most commonly affected in rheumatic heart disease, the late sequelae of acute rheumatic fever. Mitral stenosis is the classic valvular complication associated with rheumatic heart disease, which is noted to have a low-pitched diastolic rumble, possibly with opening snap early in the course of the disease.

How well did you know this?
1
Not at all
2
3
4
5
Perfectly
94
Q

pansystolic (holosystolic), blowing in quality, best heard at the apex, and often radiating to the left axilla

A
  • Mitral regurgitation
  • pansystolic (holosystolic), blowing in quality, best heard at the apex, and often radiating to the left axilla
  • The murmur can increase in intensity with handgrip or squatting
  • Papillary muscle rupture/endocarditis
How well did you know this?
1
Not at all
2
3
4
5
Perfectly
95
Q

Cullen and Grey-Turner signs

A

Acute pancreatitis

How well did you know this?
1
Not at all
2
3
4
5
Perfectly
96
Q

Medial ankle ulcer

A

venous insufficiency

How well did you know this?
1
Not at all
2
3
4
5
Perfectly
97
Q

Macerated debris in the auditory canal

A

otitis externa.

How well did you know this?
1
Not at all
2
3
4
5
Perfectly
98
Q

Murphy sign

A

cholecystitis

How well did you know this?
1
Not at all
2
3
4
5
Perfectly
99
Q

A 52-year-old man had abdominal surgery to remove a portion of his stomach. He subsequently developed a megaloblastic anemia. A deficiency in which of the following substances is the most likely cause of this patient’s anemia?

A

Intrinsic factor

Intrinsic factor is a glycoprotein produced by parietal cells of the stomach, and it is required for the absorption of vitamin B12 (cobalamin) in the terminal ileum. Gastrectomy can result in insufficient intrinsic factor being produced, and predisposes individuals to vitamin B12 deficiency, which can lead to megaloblastic anemia.

How well did you know this?
1
Not at all
2
3
4
5
Perfectly
100
Q

___________act on the sodium-potassium-2 chloride cotransporter (symporter) in the thick ascending limb of the loop of Henle. They inhibit the reabsorption of one sodium, one potassium, and two chloride ions.

A

Sodium-potassium-2chloride (Na+-K+-2Cl-) cotransporter

Loop diuretics act on the sodium-potassium-2 chloride cotransporter (symporter) in the thick ascending limb of the loop of Henle. They inhibit the reabsorption of one sodium, one potassium, and two chloride ions.

How well did you know this?
1
Not at all
2
3
4
5
Perfectly
101
Q

An abdominal radiograph shows a double-bubble sign

infant’s vomit as bilious

A

Duodenal atresia

Duodenal atresia is a congenital defect in the duodenum, resulting in a partial or complete obstruction. It is associated with Down syndrome. Infants will have bilious vomiting and abdominal distension. It is usually confirmed with an abdominal x-ray, which shows both air in the stomach and the duodenum; hence double bubble. Treatment involves surgical correction.

How well did you know this?
1
Not at all
2
3
4
5
Perfectly
102
Q

6-month-old boy with known Tetralogy of Fallot is brought in by his mother for a “Tet spell.” Which of the following class of medication is best used to treat a “Tet Spell?”

A

Phenylephrine

Patients with Tetralogy of Fallot (in addition to other cardiac defects) have varying degrees of right ventricular outflow tract obstruction. The larger amount of blood that is shunted from the right ventricle directly to systemic circulation, the less the amount of blood that is oxygenated.

“Tet spells” are presumed to be due to acute increase in pulmonary vascular resistance, clinically manifesting as acute episodes of systemic cyanosis followed by syncope, which may result in hypoxia and death. The treatment of a “tet spell” is to increase systemic vascular resistance to allow for temporary reversal of the shunt. Phenylephrine is a vasopressor that increases systemic vascular resistance.

How well did you know this?
1
Not at all
2
3
4
5
Perfectly
103
Q

With a suspected diagnosis of giant cell arteritis, what other condition is also likely to be present in this patient?

A

Polymyalgia rheumatica can be associated with temporal arteritis in 50% of cases.

How well did you know this?
1
Not at all
2
3
4
5
Perfectly
104
Q

A 47-year-old man comes to the emergency department because of sudden onset abdominal pain that started one hour ago. He states that he has had epigastric pain off and on over the past two years. He describes that pain as a gnawing and burning sensation that typically occurs two hours after a meal. Temperature is 37.2°C (99°F), pulse rate is 92/min, respirations are 20/min, and blood pressure is 118/62 mmHg. Physical examination of the abdomen shows rigidity, involuntary guarding, and mild rebound tenderness. Upright chest x-ray study shows the presence of air underneath the right hemidiaphragm. Which of the following is the most appropriate interpretation of this radiographic finding?

A

Pneumoperitoneum

Upright chest x-ray study is often the initial imaging study obtained when perforated peptic ulcer is suspected. Free air under the diaphragm (pneumoperitoneum) found on an upright chest X-ray is indicative of hollow organ perforation, and mandates further work-up or exploration. Free intra-abdominal gas forms a radiolucent crescent under the diaphragm, and is separated from the lungs only by the thin membrane of the diaphragm. In the setting of an appropriate history, and peritonitis on physical examination, free air on x-ray is sufficient to justify surgical exploration.

How well did you know this?
1
Not at all
2
3
4
5
Perfectly
105
Q

Late systolic murmur with click

A

mitral valve prolapse with mitral regurgitation

How well did you know this?
1
Not at all
2
3
4
5
Perfectly
106
Q

ECG by QRS duration of > 120 milliseconds, absence of Q waves in leads I, V5 and V6, broad monophasic R waves in I, V5 and V6, and ST and T wave displacement opposite to the major deflection of the QRS complex.

A

LBBB

How well did you know this?
1
Not at all
2
3
4
5
Perfectly
107
Q

blowing, harsh, whooping, or honking holosystolic murmur heard best at the apex. These murmurs usually radiate to the axilla. The murmur can increase in intensity with handgrip or squatting

A

Mitral regurgitation

How well did you know this?
1
Not at all
2
3
4
5
Perfectly
108
Q

holosystolic murmur heard best at the apex and radiates to the left axilla.

A

Mitral regurgitation

How well did you know this?
1
Not at all
2
3
4
5
Perfectly
109
Q

deficiency is characterized by megaloblastic anemia

A

Vitamin B12

How well did you know this?
1
Not at all
2
3
4
5
Perfectly
110
Q

I, aVL, V5, and V6

A

lateral wall infarction.

How well did you know this?
1
Not at all
2
3
4
5
Perfectly
111
Q

Which of the following valves of the heart is most commonly affected in patients with rheumatic heart disease?

A

The valves most affected by rheumatic fever, in order, are the mitral, aortic, tricuspid, and pulmonary valves.

How well did you know this?
1
Not at all
2
3
4
5
Perfectly
112
Q

Beriberi

A

Thiamine (vitamin B-1) deficiency can cause beriberi, and Wernicke-Korsakoff syndrome.

How well did you know this?
1
Not at all
2
3
4
5
Perfectly
113
Q

transient monocular vision loss

A

amaurosis fugax, which is associated with temporary loss of blood flow to the retina that can be due to temporal arteritis or CVA.

How well did you know this?
1
Not at all
2
3
4
5
Perfectly
114
Q

A 35 year old male presents to the clinic for evaluation of recurrent and multiple peptic ulcers found on endoscopy, failing maximal proton-pump inhibitor therapy. The patient has no history of nonsteroidal anti-inflammatory drug use and recent negative testing of Helicobacter pylori infection. Which of the following is the most appropriate diagnostic study to order at this time?

A

Gastrin levels

The most likely diagnosis in this scenario is Zollinger Ellison syndrome (ZES). ZES is caused by hypersecretion of gastric acid due to neuroendocrine tumors (gastrinomas). ZES classically presents with recurrent peptic ulcer disease with no major risk factors. Initial evaluation for suspected ZES includes fasting serum gastrin levels. Serum gastrin levels greater than 10 times normal in the setting of a gastric pH <2 is diagnostic of ZES. In nondiagnostic serum gastrin levels, a secretin stimulation test helps distinguish gastrinomas from other causes of hypergastrinemia.

How well did you know this?
1
Not at all
2
3
4
5
Perfectly
115
Q

most common cause of restrictive cardiomyopathy

A

Amyloidosis

How well did you know this?
1
Not at all
2
3
4
5
Perfectly
116
Q

One of your adult patients presents to your clinic 2 days after a visit to the emergency room for palpitations. His ECG from the emergency room visit demonstrates a tachycardic rhythm with three different P-wave morphologies, and R-wave progression across the precordial leads. The remainder of the ECG is unremarkable. Which of the following best describes the underlying diagnosis?

A

Multifocal atrial tachycardia

Multifocal atrial tachycardia is defined as a tachycardic rhythm with at least 3 different P-wave morphologies.

How well did you know this?
1
Not at all
2
3
4
5
Perfectly
117
Q

Which of the following is the most common cause of large bowel obstruction in adults in the United States?

A

Carcinoma

Colon cancer is the most common cause of large bowel obstruction (LBO). Other causes include volvulus, diverticular disease, and adhesions, however carcinoma is responsible for around 60% of LBOs. Patients will present with abdominal pain and obstipation. Surgery is often needed.

How well did you know this?
1
Not at all
2
3
4
5
Perfectly
118
Q

Which of the following is the most common cause of peptic ulcer disease?

A

Helicobacter pylori

H. pylori is a gram negative bacteria that is responsible for over 80% of both gastric and duodenal ulcers. All patients with peptic ulcer disease should be tested for H. pylori, either through biopsy, urease breath testing, or stool antigen testing. Risk factors associated with H. pylori include low socioeconomic status, crowded living conditions, and lack of running water.

How well did you know this?
1
Not at all
2
3
4
5
Perfectly
119
Q

present with acute painless vision loss, but the retina will have the appearance of the classic ‘blood and thunder’, with diffuse retinal hemorrhage, dilated, tortuous veins, and optic disc edema.

A

Central retinal vein occlusion

Central retinal vein occlusion (CRVO) will also present with acute painless vision loss, but the retina will have the appearance of the classic ‘blood and thunder’, with diffuse retinal hemorrhage, dilated, tortuous veins, and optic disc edema.

How well did you know this?
1
Not at all
2
3
4
5
Perfectly
120
Q

Central retinal vein occlusion (CRVO) is generally caused by

A

CRVO is generally caused by thrombosis,

How well did you know this?
1
Not at all
2
3
4
5
Perfectly
121
Q

early diastolic decrescendo murmur heard best at the right upper sternal border. It is useful to have the patient sitting upright, leaning forward, and exhaling.

A

Aortic regurgitation

Maneuvers to increase intensity of the aortic insufficiency murmur include arterial occlusion, squatting, or sitting up

How well did you know this?
1
Not at all
2
3
4
5
Perfectly
122
Q

low-pitched diastolic rumble with presystolic accentuation, best heard at the apex with the patient in the left lateral decubitus position.

A

Mitral stenosis

How well did you know this?
1
Not at all
2
3
4
5
Perfectly
123
Q

Which of the following anatomical locations does Crohn’s disease most commonly affect?

A

Distal Ileum

Crohn’s disease can occur in any part of the GI tract, mouth to anus. The distal ileum is the most common location affected by Crohn’s disease, occuring in up to 80% of patients. Thus, sigmoidoscopy should not be used in evaluation of these patients. Because bile salts are normally absorbed in the distal ileum, many patients with Crohn’s disease have a secretory diarrhea caused by passage of bile salts into the colon. Vitamin B12 deficiency is also common due to impaired absorption in the ileum.

How well did you know this?
1
Not at all
2
3
4
5
Perfectly
124
Q

holosystolic murmur

A

Mitral regurgitation

How well did you know this?
1
Not at all
2
3
4
5
Perfectly
125
Q

Which of the following cardiac abnormalities is most common among patients with Systemic Lupus Erythematosus

A

Pericarditis

Pericarditis is inflammation of the fibrous pericardial sac. Pericardial effusion may also be present. Patients present with pleuritic chest pain that is worse when lying supine. On examination, a pericardial friction rub may be heard. Associated ECG findings include diffuse ST changes. Due to the inflammatory nature of SLE, pericarditis may occur in up to 66% of patients with this diagnosis. Pericarditis in patients with SLE usually follows the typical course without additional complications.

126
Q

Which of the following antibiotic regimens is indicated in the treatment of a patient with cholecystitis?

A

Ceftriaxone and metronidazole

Acute cholecystitis is inflammation of the gallbladder, usually due to cystic duct obstruction by gallstones. Patients present with right upper quadrant abdominal pain, fever, nausea, and vomiting. Look for a positive Murphy’s sign on physical examination. The most commonly associated bacteria include Escherichia coli, Enterococcus, and Klebsiella. A third-generation cephalosporin, such as ceftriaxone, plus metronidazole are recommend first line for both gram-negative and anaerobic coverage.

127
Q

Hypertension, bradycardia, irregular respirations

A

Cushing’s Triad, which describes signs associated with elevated intracranial pressure (ICP).

128
Q

diastolic rumble heard best over the (left 4th intercostal space).

A

Tricuspid stenosis

129
Q

inflammatory bowel disease most often affecting the ileum, with segmental involvement, transmural inflammation, and development of granulomas?

A

Crohn disease

Crohn disease is a chronic transmural inflammatory bowel disease that usually affects the distal ileum and colon, but may occur in any part of the gastrointestinal tract. The bowel wall is asymmetrically and segmentally affected, producing ‘skip’ areas between diseased segments. Endoscopic appearance of the mucosa shows patchy, discrete ulcerations (‘cobblestone’ appearance) separated by normal appearing mucosa. Epitheliod granulomas may also be detected in the bowel wall or lymph nodes, and are pathognomonic for this condition. Symptoms include diarrhea, and abdominal pain. Abscesses, internal and external fistulas, and bowel obstruction may also occur. In addition, extraintestinal symptoms, particularly arthritis, may also be present.

130
Q

unilateral sensorineural hearing loss but would be associated with disequilibrium.

A

Eighth cranial nerve palsy

Eighth cranial nerve palsy would result in unilateral sensorineural hearing loss but would be associated with disequilibrium.

131
Q

Which of the following organisms is the most common cause of viral myocarditis?

A

Parvovirus

132
Q

mid-diastolic rumbling murmur following an opening snap.

A

Mitral stenosis

133
Q

Rales or crackles in the lung base, bilaterally

A

CHF lung sounds

134
Q

Cholera tx

A

Ciprofloxacin

Bactrim and fluoroquinolones are indicated in therapy of Shigella and Cholera.

135
Q

A 56-year-old man is brought to the emergency department by ambulance because of altered mentation and confusion. Physical examination shows hyperreflexia, cerebellar ataxia, and asterixis. You also notice a musty breath odor. Administration of which of the following medications is most appropriate for this patient’s underlying condition?

A

Lactulose

This patients has a history consistent with hepatic encephalopathy. The primary toxin associated with this condition is ammonia. Lactulose functions to lower levels of ammonia in the blood.

136
Q

Reynaud’s treatment

A

Nifedipine or amlodipine

The patient’s symptoms of pale and cyanotic fingers when presumably in the cold, is consistent with Raynaud’s phenomenon. This disorder is caused by vasoconstriction of cutaneous arterioles. Calcium channel blockers, such as nifedipine or amlodipine, inhibit vasospasms and have vasodilatory effects. They are considered the first-line therapy for Raynaud’s phenomenon. The patient should also avoid the cold or wear gloves and quit smoking.

137
Q

Which of the following Vitamin deficiencies is most likely to present with poor wound healing?

A

Vitamin C

Scurvy is the condition of Vitamin C deficiency. Lack of vitamin C prevents proper synthesis of collagen and connective tissues. It presents with poor wound healing, and may also show petechiae, arthralgias, bleeding gums, and loose teeth. Onset of symptoms is within several months of following a diet deficient in Vitamin C.

138
Q

dilated loops of bowel with air-fluid levels.

A

SBO

139
Q

ulcers are painful immediately after, or during a meal.

A

Gastric ulcers are painful immediately after, or during a meal.

gastric==gorge

140
Q

streptococcal tonsillitis/pharyngitis tx?

A

Penicillin

141
Q

An 83-year-old man comes to the emergency department complaining of abdominal pain and blood in his stool. He states that he has been feeling nauseated but had just one episode of vomiting. He is currently running a low grade fever. He is diaphoretic and tachycardic. Physical examination reveals that the pain is localized in the left lower quadrant. Which of the following is the next best step in the evaluation of this patient?

A

CT scan with contrast

This patient is suspected to have diverticulitis. Abdominal CT with oral and IV contrast is the best study to perform. The contrast allows you to visualize the anatomy and other complications much better than a CT without contrast. On the CT scan, look for bowel wall thickening, fat stranding, and diverticula.

142
Q

A 71-year-old man comes to the office because of progressively worsening exertional dyspnea and fatigue over the past year. He occasionally experiences a “forceful” heartbeat, but denies chest pain, dizziness or syncope. Blood pressure is 140/40 mmHg. Pulses are bounding. Palpation of the precordium shows laterally displaced point of maximal impulse. Auscultation of the heart shows a grade III/VI diastolic murmur at the lower left sternal border, and a S3 gallop. The lungs do not have any adventitious sounds. There is no peripheral edema. An electrocardiogram demonstrates left ventricular hypertrophy. Which of the following is the most likely diagnosis?

A

Aortic valve regurgitation

Aortic regurgitation (AR) or insufficiency is due to incompetence of the aortic valve resulting in backflow of blood from the aorta into the left ventricle. It may be congenital (e.g. bicuspid valve) or acquired (e.g. endocarditis or collagen vascular disease), and can occur acutely or chronically. Patients with chronic AR typically present with dyspnea, chest pain, and an uncomfortable awareness of the heartbeat and palpitations (often described as a “forceful” heartbeat). On physical examination patients often have a widened pulse pressure (the difference between the systolic and diastolic pressures). This is due to an elevated stroke volume during systole as well as a significant fall in diastolic pressure within the aorta due to the incompetent aortic valve. Diastolic pressures are often lower than 60 mmHg with pulse pressures often exceeding 100 mmHg. Patients may have a Corrigan (“water hammer”) pulse, and a bobbing motion of the head with each heartbeat. A diastolic murmur is characteristic. A S3 gallop may be present if left ventricular dysfunction is present. Chronic aortic regurgitation leads to left ventricular hypertrophy, which can be manifested by a laterally displaced point of maximal impulse and anbormal EKG findings.

143
Q

fever, leukocytosis and RUQ pain.

A

Cholecystitis

Cholecystitis is infection of the gallbladder. Patients may also have fever, leukocytosis and RUQ pain. Gallbladder inflammation and surrounding inflammatory fluid around the gallbladder is generally noted on abdominal ultrasound.

144
Q

pyloric stenosis is nonbilious vomiting. true or false?

A

true; IT IS NONBILIIOUS

145
Q

What is the test of choice for a patient with a suspected blowout fracture?

A

CT of the orbits

CT scan of the orbits is the best test to diagnose an orbital blowout fracture and can also show important associated injuries such as extraoccular muscle entrapment, orbital hematoma or globe rupture.

146
Q

A 44-year-old woman presented to the emergency department with a one day history of fever, and three day history of progressively worsening right upper quadrant pain, and nausea. Diagnostic studies show leukocytosis with a left shift, gallstones with gallbladder wall thickening, and sonographic Murphy sign. Acute calculous cholecystitis is diagnosed. Which of the following is the most appropriate initial antibiotic therapy for this patient’s condition?

A

Ceftriaxone

In otherwise healthy patients with community-acquired acute calculous cholecystitis of mild-to-moderate severity, recommended initial antibiotic treatment includes cefazolin, cefuroxime, or ceftriaxone. Mild severity is often defined as no organ dysfunction, and mild inflammatory changes of the gallbladder. Moderate severity is often defined as the presence of leukocytosis (>15,000~18,000 cells/mm3), symptom duration >72 hours, palpable tender mass in the right upper quadrant, and marked local inflammation on imaging studies.

147
Q

A 27-year-old woman comes to your office with upper respiratory infection symptoms. On otoscopic exam of the left ear you visualize straw-colored fluid behind the tympanic membrane and prominent landmarks. The right ear appears normal. You suggest conservative management of her URI. At a followup visit 3 months later, she states her URI symptoms have resolved, but complains of some hearing loss in the left ear. Her otoscopic findings are unchanged. What is the next best step in the evaluation of this patient?

A

Refer to ENT

The otoscopic findings in this patient suggest serous otitis media. This condition is less common in adults, and typically occurs following an URI, chronic allergic rhinitis, or barotrauma. Adults with persistent unilateral serous otitis media longer than 12 weeks should be referred to ENT for further management with possible myringotomy +/- tube placement.

148
Q

A mother presents to your pediatric clinic with her 2 month old female infant. The baby is being breastfed. Mom informs you that she has noticed white areas on the inside of her child’s mouth. Upon exam you note white plaques on the buccal mucosa that scrape off with the tongue blade revealing a erythematous base. What is the most appropriate treatment plan?

A

Miconazole applied topically

The presentation is consistent with thrush and requires anti-funal treatment. Miconazole gel is the pharmacologic therapy of choice, but Nystatin may be used as well. Additionally, the mother needs to be evaluated and treated.

149
Q

most common site of obstruction from aspiration of small objects (e.g. peanut)

A

Right mainstem bronchus

The right mainstem bronchus is the most common site of obstruction from aspiration of small objects (e.g. peanut). This results from the anatomy of the bronchial tree favoring the right side (larger diameter and more vertical orientation). The metallic foreign body shown in the x-ray lies cephalad to the carina.

150
Q

esophageal varices tx?

A

Octreotide

This medication causes vasoconstriction and is indicated in the treatment of esophageal varices. All other listed medications could be harmful to the patient and are not indicated in the therapy of esophageal varices.

151
Q

most commonly occluded of the coronary arteries.

A

The LAD artery is the most commonly occluded of the coronary arteries.

Anterior left ventricle and interventricular septum

The left anterior descending artery (LAD) and its branches generally supplies most of the interventricular septum, the anterior, lateral, and apical wall of the left ventricle, most of the right and left bundle branches, and the anterior papillary muscle of the mitral valve.

152
Q

emergent DVT tx

A

Heparin

Heparin is a blood thinner that is used as initial therapy in deep venous thrombosis. It inactivates thrombin and factor Xa, thereby inhibiting clotting ability and preventing the existing clot from enlarging.

153
Q

Low-pitched diastolic rumble, possibly with opening snap

A

mitral stenosis

The mitral valve is most commonly affected in rheumatic heart disease, the late sequelae of acute rheumatic fever. Mitral stenosis is the classic valvular complication associated with rheumatic heart disease, which is noted to have a low-pitched diastolic rumble, possibly with opening snap early in the course of the disease.

154
Q

Complications of ___________include: ascites, spontaneous bacterial peritonitis, splenomegaly, esophageal varices, hemorrhoids, portosystemic shunting, and caput medusae.

A

portal hypertension

155
Q

ulcers are painful a few hours after a meal, when acid is secreted without food to buffer it

A

duodenal ulcers are painful a few hours after a meal, when acid is secreted without food to buffer it.

duodenal==delayed

156
Q

initial stretching of the heart prior to contraction

A

Preload

157
Q

deficiency include anorexia, muscle cramps, paresthesias, and irritabiltiy. Advanced deficiency affects the cardiovascular system or nervous system.

A

Thiamine

Vitamin B1

158
Q

It is characterized as a foreign body sensation with photophobia and pain. There can also be an associated decreased visual accuity.

A

Ultraviolet keratitis

Ultraviolet keratitis is an effect of the eye being exposed to ultraviolet rays for significant duration of time. It is characterized as a foreign body sensation with photophobia and pain. There can also be an associated decreased visual accuity.

159
Q

TX for acute otitis MEDIA

A

Prescribe oral amoxicillin, and schedule a follow up visit for two weeks to make sure infection has cleared

Otitis media is characterized by a painful ear, fever, and hearing impairment. On physical examination, a bulging, erythematous tympanic membrane will be present, with reduced mobility. Otorrhea is usually only present if the tympanic membrane has ruptured.

160
Q

results in acute vision loss, but typically the patient experiences pain. It is caused by the rapid narrowing of the anterior chamber angle, thereby preventing drainage of aqueous fluid through the trabecular meshwork. This is an ophthalmic emergency. Patients will have a red, painful eye and fixed mid-dilated pupil. Nausea and vomiting can also occur.

A

Acute angle closure glaucoma

Acute angle closure glaucoma results in acute vision loss, but typically the patient experiences pain. It is caused by the rapid narrowing of the anterior chamber angle, thereby preventing drainage of aqueous fluid through the trabecular meshwork. This is an ophthalmic emergency. Patients will have a red, painful eye and fixed mid-dilated pupil. Nausea and vomiting can also occur. Emergent evaluation by an ophthalmologist is critical.

161
Q

A 43-year-old male complains of right eyelid and surrounding area swollening, redness, and pain since yesterday. He says he was seen three days ago and diagnosed with sinusitis. Despite antibiotics, he has not been improving. His gaze is restricted in all directions and vision scores are 20/20 OS, 20/80 OD. His temperature of 102.5 F (39.2 C). What is the best plan for this patient?

A

Admit to the hospital for IV antibiotics

This patient presentation is concerning for periorbital cellulitis. He has failed out patient antibiotic therapy and is worsening. He should therefore be admitted to the hospital for IV antibiotics. The other listed options are not aggressive enough given this patient’s presentation.

162
Q

Rib notching

A

coarctation of the aorta. Coarctation of the aorta is a narrowing in the proximal thoracic aorta. Classically, patients have hypertension in the upper extremities, with hypotension in the lower extremities, and femoral delay of pulses. Rib notching of the posterior ribs occurs due to erosion from collateral arteries. An enlarged aortic notch may also be seen.

163
Q

Endocarditis tx

A
  1. Antibiotic therapy for 6 weeks is recommended in the treatment of infective endocarditis. Staph aureus is the most common etiologic agent, although streptococcus and enterococcus are also common. Although ideal antimicrobial coverage should be tailored to culture sensitivities, empiric antibiotic coverage should cover MRSA, strep and enterococcus (e.g. vancomycin and third-generation cephalosporin).
  2. In a non-IV drug user the most likely etiologic agent is Strep viridans and length of treatment is usually 4 weeks. Again, while ideal antimicrobial coverage should be tailored to culture sensitivities, empiric antibiotic coverage should cover MRSA, strep, and enterococcus.
164
Q

Which of the following lesions of the colon has the greatest risk of malignancy?

A

Villous adenoma

Adenomatous polyps are the typical polyp most commonly categorized as neoplastic polyps, and they account for about two-thirds of all colonic polyps. Most colorectal cancers arise from these polyps. Although several distinct pathological types exist, adenomatous polyps are commonly classified as tubular, tublovillous, and villous. Villous adenomas account for 5-15% of adenomas. Several factors determine risk for focal cancer within an individual adenoma, however, among those are villous histology, increasing polyp size, and high grade dysplasia.

165
Q

combination of anxiety, hypertension, and an elevated urine metanephrine level

A

Pheochromocytoma

166
Q

A 14 year old female is brought to your clinic by her mother with a complaint of a bump on her right eyelid times two days. She admits to pain, light sensitivity, lid heaviness, and increased tearing in the affected eye. Her vision is unaffected. On physical exam there is a small, erythematous mass that is tender to the touch. What is the most likely diagnosis?

A

Hordeolum

This is classic presentation of a hordeolum, or stye.

167
Q

typically occurs in children between ages 1-8 years, and is characterized by prolonged fever, exanthem, conjunctivitis, mucous membrane inflammation, bright red ‘strawberry tongue’, erythema of the palms and soles, and lymphadenopathy. Coronary artery aneurysms may develop and rupture, or cause myocardial infarction.

A

Kawasaki disease

Kawasaki disease (KD) is a vasculitis of medium-sized arteries, most significantly the coronary arteries, which are involved in about 20% of untreated patients. KD typically occurs in children between ages 1-8 years, and is characterized by prolonged fever, exanthem, conjunctivitis, mucous membrane inflammation, bright red ‘strawberry tongue’, erythema of the palms and soles, and lymphadenopathy. Coronary artery aneurysms may develop and rupture, or cause myocardial infarction. Diagnosis is by clinical criteria.

168
Q

ECG by QRS duration of > 120 milliseconds, rsR’ “bunny ear” pattern in the anterior precordial leads (leads V1-V3), slurred S waves in leads I, aVL and frequently V5 and V6.

A

R BBB

169
Q

A 16 year old male presents to the emergency department after being struck in the left eye with a baseball during practice. He is complaining of pain in and around the eye and some double vision. What physical exam finding would most raise your suspicion for a blowout fracture?

A

Restricted upward gaze

Restricted upward gaze due to entrapment of eye muscles would be most likely to raise your suspicion of a fracture.

170
Q

A 65-year-old woman comes to the emergency department for right upper quadrant abdominal pain and fever for 1 week. Her blood pressure is 88/54 mmHg, her pulse is 109 and regular, her respiratory rate is 22, temperature is 102.5F, and her oxygen saturation is 99% on room air. Physical examination demonstrates a mildly lethargic and moaning patient with moderate tenderness upon palpation of the right upper quadrant of the abdomen with guarding but no peritonitis. There is no pain elicited with costovertebral angle percussion. She demonstrates scleral icterus and diffusely jaundiced skin. Laboratory tests, abdominal ultrasound, and a CT abdomen/pelvis are ordered and pending. Which of the following is her most likely diagnosis?

A

Ascending cholangitis

This patient is exhibiting signs of Reynolds Pentad which includes right upper quadrant pain, jaundice, fever, hypotension, and altered mental status. This constellation is consistent with ascending cholangitis.

171
Q

Generally a painless process that may result in partial vision loss or floaters.

A

Vitreous hemorrhage

172
Q

Which of the following Vitamin deficiencies presents with the 3 D’s (diarrhea, dermatitis, and dementia)?

A

Niacin

This condition is known as Pellagra, and manifests itself in Niacin deficiency.

Vitamin B3

173
Q

A 3-year-old boy is brought to the office by his mother because of fever, rash, watery red eyes, and redness and swelling of his hands and feet for the past four days. Physical examination shows patchy generalized macular erythema, bilateral bulbar conjunctival injection, erythema, fissuring and crusting of the lips, strawberry-colored tongue, and mild erythema and edema of the palms and soles with some desquamation of the finger and toe tips. Which of the following is the most likely diagnosis?

A

Kawasaki disease

Kawasaki disease (KD) is a vasculitis of medium-sized arteries, most significantly the coronary arteries, which are involved in about 20% of untreated patients. KD typically occurs in children between ages 1-8 years, and is characterized by prolonged fever, exanthem, conjunctivitis, mucous membrane inflammation, bright red ‘strawberry tongue’, erythema of the palms and soles, and lymphadenopathy. Coronary artery aneurysms may develop and rupture, or cause myocardial infarction. Diagnosis is by clinical criteria.

174
Q

A 55 year old male presents to the clinic for evaluation of chronic pruritis. He also states that he has had a ten pound weight loss over the last two months and feels tired all of the time. Physical exam reveals abdominal fluid retention as well as jaundiced skin, redness of his palms and spider angiomas on his back. Patient admits to chronic overuse of alcohol. You suspect that this patient has Hepatocellular carcinoma secondary to cirrhosis. Which of the following tumor markers would you order to help confirm the suspected diagnosis?

A

Alpha-fetoprotein

This is the tumor marker you would expect to be elevated with liver cancer.

175
Q

Test results come across your desk for a 32 year old male patient that was seen in your clinic last week. He tested serum positive for Hepatitis D. Which other co-infection is likely in this patient?

A

Hepatitis B

Hepatitis D can not survive unless Hepatitis B is also present. The patient must also be Hepatitis B positive. Hepatitis D infection can range from asymptomatic to fulminant liver failure. It is endemic to the Mediterranean area, but is rare in the United States, mostly occuring in high risk groups.

Dick needs Boobs

176
Q

common causes of _____epistaxis are dry mucosa, nose picking, foreign body, and mucosal edema from rhinitis.

A

anterior

177
Q

A patient presents with visual field loss of the lateral portion of each eye. What is the most likely location for the lesion?

Bitemporal hemianopsia

A

Optic chiasm

Bitemporal hemianopsia is caused by a lesion at the optic chiasm.

178
Q

“rabbit ears” in V1

A

R BBB

179
Q

Mobile “olive” mass in the epigastrum

A

infant with pyloric stenosis

180
Q

Campylobacter jejuni tx

A

Erythromycin

Macrolides can be used in the therapy of a patient with Campylobacter jejuni.

181
Q

anterior epistaxis

A

Kiesselbach’s plexus

Around 90% of nosebleeds arise from Kiesselbach’s plexus (an area of the anterior nasal septum where three main vessels converge). The mainstays of treatment in epistaxis include direct pressure for 15 minutes, topical sympathomimetics, and nasal tamponade methods.

182
Q

“water bottle heart”

A

pericardial effusion.

183
Q

9-month-old boy

sausage-shaped mass in the right mid-abdomen

. Ultrasound study of the abdomen shows target sign

A

intussusception.

tx:

Therapeutic enema

This patient has intussusception. Barium enema was at one point in time, the preferred initial diagnostic study as it was both diagnostic, and therapeutic. Unfortunately, barium occasionally enters the peritoneum through a clinically unsuspected perforation, and may cause significant peritonitis. Currently, ultrasonography is the preferred means of diagnosis, and the classic finding is that of a target sign. Once the diagnosis is confirmed, most often an air enema is used for reduction. This lessens the likelihood of bowel perforation.

184
Q

Which of the following cardiomyopathies has diastolic dysfunction as a hallmark finding?

A

Restrictive cardiomyopathy

Diastolic dysfunction is a hallmark of restrictive cardiomyopathy. It is caused by infiltration or fibrosis of the ventricular walls. Ventricular contractility is relatively preserved, but the diastolic filling phase is impeded.

185
Q

eating contaminated meat or eggs. Symptoms include low-grade fever and diarrhea. The incubation period is 6-48 hours.

A

salmonella

186
Q

posterior epistaxis

A

Woodruff’s plexus

Woodruff’s plexus, of which the sphenopalatine artery is the major branch, is located posteriorly and the most common site of posterior epistaxis. Posterior epistaxis is more difficult to control and may require consultation and management by ENT.

187
Q

right homonymous hemianopsia.

A

Left optic tract

A left optic tract lesion would cause right homonymous hemianopsia.

188
Q

Systolic ejection murmur heard best over the 2nd and 3rd intercostal space left of the sternum

A

pulmonary stenosis.

189
Q

loss of the haustral marking of the colon.

A

Ulcerative colitis

Patients with ulcerative colitis with have recurrent episodes of abdominal pain and bloody diarrhea. Inflammation starts in the rectum and spreads proximally in a circumferential, continuous pattern, without skip-lesions. It can involve the entire colon. On imaging, loss of haustral markings may be noted, along with narrowing of the lumen.

190
Q

Factors contributing to _______epistaxis include anticoagulant therapy, hemophilia, aneurysm of the carotid artery, and neoplasm.

A

posterior

191
Q

A 2-year-old boy is brought to the office by his father for evaluation of painful mouth sores for the past two days. Four days ago, the child began having general malaise, anorexia, and sleeplessness. He also developed a fever with a temperature up to 38.0°C (100.4°F). Pain associated with the mouth lesions prevents the boy from wanting to eat or drink. Physical examination of the oropharynx shows several clusters of small vesicles with erythematous bases and areas of ulceration, both of which bleed easily, on the lips, buccal mucosa, tongue, gingiva and hard palate. Which of the following is the most likely diagnosis?

A

Gingivostomatitis

The diagnosis of gingivostomatitis (herpes simplex virus type 1) is generally made clinically, based upon the typical appearance and location of oral and extraoral lesions. The enanthem begins with red, edematous marginal gingivae that bleed easily and clusters of small vesicles. The vesicles become yellow after rupture and are surrounded by a red halo. They coalesce to form large, painful ulcers of the oral and perioral tissues. They bleed easily and may become covered with a black crust. The lesions involve the buccal mucosa, tongue, gingiva, hard palate, and pharynx; the lips and perioral skin are affected in approximately two-thirds of cases. Mild lesions typically heal without scarring in about a week, but healing may require 14 to 21 days in severe cases.

192
Q

crescendo-decrescendo murmur over 2nd right intercostal space, radiating into the neck.

A

Aortic stenosis

193
Q

red, jelly-like stool

A

Intussusception

Intussusception is telescoping of the intestines, and most commonly occurs in children ages 6 months to 3 years. The classic triad of this disease is abdominal pain, vomiting, and currant jelly stools. Sometimes a sausage-shaped mass can be palpated on examination. Look for a “bull’s eye” image on abdominal ultrasound. Treatment involves reduction with hydrostatic or pneumatic enemas, or in severe cases, surgery.

194
Q

Allergic rhinitis, atopic dermatitis, asthma

A

This represents the atopic triad. Allergic rhinitis, atopic dermatitis, and asthma are all associated with allergic hypersensitivity reactions, and often occur together.

195
Q

low-frequency diastolic murmur heard best at the apex.

A

Mitral stenosis

196
Q

unilateral sensorineural hearing loss. WITHOUT disequilibrium

A

Acoustic neuroma

197
Q

Anti-centromere antibodies

A

Scleroderma

198
Q

sudden painful vision loss. Visual inspection of the eye may reveal a mid-dilated, fixed pupil and corneal haziness.

PAINFUL

A

Acute angle glaucoma

Patients with acute angle glaucoma present with sudden painful vision loss. Visual inspection of the eye may reveal a mid-dilated, fixed pupil and corneal haziness.

199
Q

slowly enlarging, painless eyelid nodules.

A

Chalazion

Chalazion are slowly enlarging, painless eyelid nodules.

200
Q

An indirect hernia is most likely to pass through which of the following structures?

A

Inguinal canal

Indirect inguinal hernias arise lateral and superior to the course of the inferior epigastric vessels (lateral to Hesselbach triangle), and then protrude through the deep (internal) inguinal ring into the inguinal canal. It passes inferomedially to emerge via the superficial ring and, if large enough, extends into the scrotum.

201
Q

Enlarged/ prominent aortic knob

A

patent ductus arteriosis

202
Q

development of heart failure late in pregnancy or in the early post partum period, with left ventricular systolic dysfunction, and absence of another identifiable cause. The etiology is not well understood, but is thought to be multifactorial.

A

Peripartum cardiomyopathy

203
Q

Which of the following is the most common cause of the development of esophageal varices in developed countries?

A

Cirrhosis

Cirrhosis, tissue damage to the liver, which is usually caused by alcohol use or hepatitis is the leading cause of portal hypertension in developed countries. Portal hypertension develops with there is resistance to portal blood flow and leads to development of esophageal varices, splenomegaly, development of abdominal wall collateral circulation.

204
Q

In a patient with a typical coronary artery circulation, which area of the heart musculature is most likely to become ischemic if a thrombus occludes the left anterior descending branch of the left coronary artery?

A

Anterior left ventricle and interventricular septum

The left anterior descending artery (LAD) and its branches generally supplies most of the interventricular septum, the anterior, lateral, and apical wall of the left ventricle, most of the right and left bundle branches, and the anterior papillary muscle of the mitral valve. The LAD artery is the most commonly occluded of the coronary arteries.

205
Q

transmural skip lesions along the GI tract from mouth to anus?

A

Crohn’s disease

206
Q

A constant PR interval >0.2 seconds

A

first degree AV block.

207
Q

Holosystolic murmur heard best at the apex

A

mitral regurgitation.

208
Q

A 34 year old female presents with right sided abdominal pain for the last three days. She has now come into the emergency room because she can’t control the pain anymore on her own. Vital signs are BP 156/87, P 102, RR 16, T 99.0F, O2Sat 99% room air. Abdominal exam reveals moderate right upper quadrant tenderness. The patient winces and halts her inspiratory breath while your hand is on her right upper quadrant under her costal margin. Her lab work reveals a total bilirubin of 1.0, ALT of 75, AST of 120, and a mildly elevated alkaline phosphatase. Which of the following is the next best step in management?

A

Right upper quadrant ultrasound

This patient clinically presents with acute cholecystitis, with a positive Murphy’s sign. A right upper quadrant ultrasound is indicated to confirm the diagnosis, ultimately with surgical consultation for admission.

209
Q

radiation to the axilla

A

Mitral valve regurgitation

Fatigue, exertional dyspnea, and orthopnea are the most prominent symptoms in chronic, severe mitral regurgitation (MR). A holosystolic murmur with radiation to the axilla is common. A S3 gallop may also be present with severe MR.

210
Q

diastolic decrescendo blowing murmur

A

Aortic regurgitation

211
Q

deficiency presents with areflexia, gait disturbances, decrease proprioception and vibratory sensation and paralysis of the eye muscles.

A

Vitamin E

212
Q

True or false: Crohn’s disease can cause mouth ulcers?

A

True: Causes oral ulcers

This is characteristic of Crohn’s disease. Patients can have multiple painful aphthous ulcers of the mouth and gums.

213
Q

A 25-year-old man who is otherwise healthy comes to the office because of persistent frequent bowel movements, intermittently with bloody mucus, for the past two weeks. He states that he has been having 2-3 bowel movements per day, and often has the feeling of needing to empty his bowels. He denies any recent illnesses or foreign travel. He takes no medications. Physical examination shows mild tenderness of the lower left abdominal quadrant without guarding or rebound. Which of the following is the most likely diagnosis?

A

Ulcerative colitis

Ulcerative colitis (UC) is a chronic inflammatory and ulcerative disease arising in the colonic mucosa, most often characterized by bloody diarrhea. Colonic involvement is usually left-sided, and most often involves the rectosigmoid area. The bowel wall is symmetrically, and uniformly involved from the rectum proximally. Fistulas and epithelioid granulomas do not occur. Extraintestinal symptoms, particularly arthritis, may also occur. Patients with UC are also at increased risk for colon cancer.

214
Q

quadrantanopsia

A

Optic radiation

A lesion at the optic radiation would cause quadrantanopsia, a loss of a quarter of the field of vision.

215
Q

Papillary muscle rupture is most common with an ______________ MI.

A

Papillary muscle rupture is most common with an inferior MI.

216
Q

A 27-year-old mildly obese woman comes to the clinic complaining of ongoing flatulence and bloating for several months. She states that her stools are very foul smelling. She says this is usually worse when eating foods that contain wheat. She then goes on to state that she wouldn’t have come in to be evaluated except she has now noticed a blister-like rash over the trunk of her body. It is very pruritic in nature. Which of the following dermatologic conditions is most likely occurring in this patient?

A

Dermatitis herpetiformis

This patient’s history is consistent with a probable diagnosis of Celiac disease. Dermatitis herpetiformis is the most common dermatologic manifestation associated with this condition. It is an autoimmune rash with very pruritic papules and vesicles, usually on the arms, knees, buttocks, or scalp. Lesions occur in groups. Treatment involves a gluten-free diet and dapsone.

217
Q

Mid-systolic click heard best at the apex

A

mitral valve prolapse.

218
Q

Which of the following antibody titers would you expect to be elevated in a patient with Primary Biliary Cirrhosis?

A

Anti-mitochondrial antibody

95% of patients with a diagnosis of PBC will have elevated AMA levels. Primary Biliary Cirrhosis most often occurs in middle-aged women, and is a chronic liver disease of insidious onset. Small intrahepatic ducts are destroyed by autoimmune cells and cholestasis results. PBC most commonly presents with asymptomatic elevation of alkaline phosphatase levels, but may present early on with generalized fatigue and pruritis. Xanthomatous lesions may occur later in the course of this disease, once cholesterol levels are sufficiently elevated. Liver biopsy is used for staging.

219
Q

continuous machine like murmur throughout both systole and diastole

A

Patent ductus arteriosus

220
Q

Beta-2-microglobulin marker

A

Multiple Myeloma and Lymphoma.

221
Q

“diamond shaped” murmur

A

aortic stenosis

222
Q

holosystolic murmur that increases with inspiration.

A

Tricuspid valve regurgitation

Tricuspid valve regurgitation is most often caused by secondary hemodynamic load on the right ventricle, but may be due to a valve abnormality (e.g. infective endocarditis, rheumatic fever, carcinoid syndrome or pulmonary hypertension). Patients present with symptoms of right-sided heart failure with increased jugular venous pressure, abdominal ascites, and lower extremity edema. They may have a parasternal lift due to right ventricular enlargement. They will have a holosystolic murmur that increases with inspiration.

223
Q

“sandpaper rash”

A

This is the description of the ‘sandpaper’ rash associated with Scarlet Fever, which is seen in infection with Group A beta-hemolytic strep.

A fine erythematous, papular rash with a rough texture generally located on the torso

224
Q

initial mainstay of therapy for hypertrophic cardiomyopathies.

A

Labetolol

Beta blockers are the initial mainstay of therapy for hypertrophic cardiomyopathies.

225
Q

TX for acute otitis EXTERNA

A

Prescribe ciprofloxacin/dexamethasone ear drops, and counsel patient to keep the ear canal dry for at least a week

226
Q

Machine-like, rough continuous murmur

A

patent ductus arteriosus.

227
Q

typically presents with a headache, unilateral or sometimes bilateral swelling, fever, and inability to move the involved eye.

A

Cavernous sinus thrombosis

Cavernous sinus thrombosis typically presents with a headache, unilateral or sometimes bilateral swelling, fever, and inability to move the involved eye.

228
Q

You are rounding on one of your patients that was admitted to the hospital over the weekend for shortness of breath and chest pain. His electrocardiogram this morning shows inverted P-waves in the inferior leads. Which of the following would be this patient’s most likely rhythm?

A

Junctional rhythm

When signals for the heart to contract originate from the AV junction, the P-waves on ECG tend to be inverted due to retrograde depolarization of the atria. Inverted P-waves should make you think Junctional rhythm first.

229
Q

Aphthous ulcers tx

A

The lesions are usually self-limiting, but topical triamcinolone oral paste can be used to accelerate healing.

230
Q

A 19-year-old woman with suspected drug overdose is undergoing prolonged gastric suctioning. This patient is at risk for which acid-base disorder?

A

Metabolic Alkalosis

Metabolic alkalosis has an elevated pH, elevated bicarbonate, and compensatory elevated carbon dioxide. It can be caused by vomiting, nasogastric tube suctioning, diuretics, and mineralocorticoid excess. Removal of HCl during gastric suctioning is the primary concern in this patient that leads to metabolic alkalosis.

231
Q

CA-125

A

ovarian cancer.

232
Q

A 35-year-old woman comes to the office for evaluation of intermittent hearing loss over the past 12 months. Physical examination shows lateralization to the right ear with Weber test, and bone conduction greater than air conduction in the right ear with Rinné test. Which of the following is the most likely explanation for these findings?

A

Conductive hearing loss of the right ear

This is characterized by lateralization to the right ear with Weber test, and bone conduction > air conduction of the right ear with Rinné test.

233
Q

A 52-year-old woman returns to your clinic for follow up after recovering from a recent viral illness. She comes in with increasing fatigue, decreased exercise tolerance and chest discomfort. Temperature is 98.7, blood pressure is 128/72 mmHg, pulse is 67 beats per minute, and respirations are 14 breaths per minute. Physical examination reveals a muffled S1 heart sound, peripheral edema, and jugular venous distention. Electrocardiogram reveals non-specific ST and T wave changes. Chest x-ray is without cardiopulmonary changes. Which of the following is the most likely diagnosis?

A

Myocarditis

This is the most likely diagnosis. Up to 40% of patients with myocarditis have recently had a viral illness. The cause is most often parvovirus. This condition is diagnosed by echocardiography.

NO PERICARDIAL FRICTION RUB MENTIONED IN QUESTION STEM

234
Q

Erythema and buldging of the tympanic membrane with decreased mobility during insufflation

A

Acute otitis media

235
Q

drug of choice for peripheral arterial disease

A

Cilostazol

236
Q

infections are from waterborne shellfish. Symptoms include profuse watery diarrhea referred to as “rice water stools”. Fever is rare. The incubation period is 12-72 hours.

A

Vibrio species

237
Q

A newborn girl is diagnosed with phenylketonuria following routine screening, and confirmatory testing. When providing dietary counseling to the girl’s parents regarding use of artificial sweeteners, which of the following must the girl avoid ingesting?

A

Aspartame (Equal)

Aspartame is a dipeptide of two amino acids, aspartic acid, and phenylalanine. When digested, it is broken down into aspartic acid, phenylalanine, and a very small amount of methanol. Patients with phenylketonuria either metabolize phenylalanine, or have a limited ability to do so.

238
Q

fairly acute, painless loss of portions of vision and patients often complain of seeing floaters.

PAINLESS

A

Retinal detachment

This patient likely has a retinal detachment. Retinal detachments cause a fairly acute, painless loss of portions of vision and patients often complain of seeing floaters.

239
Q

A 64-year-old man is diagnosed with acute mitral regurgitation from an inferior wall myocardial infarction sustained one day ago. In addition to the presence of a new murmur, which of the following additional findings on physical examination is most likely to be present in this patient?

A

Pulmonary crackles

Acute left ventricular failure secondary to myocardial infarction is a cause of acute cardiogenic pulmonary edema and a hallmark of predominantly left-sided heart failure. Papillary muscle rupture is most common with an inferior MI. Symptoms may include shortness of breath, cough, and even production of pink, frothy sputum. Physical examination findings include diaphoresis, tachypnea, tachycardia, air hunger, agitation/confusion, bilateral rales and/or wheezes, S3, and possibly jugular venous distention.

An important point to remember is that even physical exam findings suggestive of right-sided heart failure can be seen in long-standing left-sided heart failure that leads to right-sided heart failure. The key here is the the patient in this scenario has acute left-sided heart failure so would not have time to develop right-sided heart failure and its physical manifestations.

240
Q

removal of an insect

A

Instillation of 2% lidocaine solution followed by insect removal

The insect should be killed prior to removal, using lidocaine (2%) or mineral oil. The insect may then be removed with use of alligator forceps or curette. Irrigation is the simplest method of foreign body removal provided the tympanic membrane is not perforated. Irrigation with water is contraindicated for soft objects, organic matter or seeds which may swell.

241
Q

earliest sign or symptom of congestive heart failure?

A

Dyspnea on exertion

242
Q

Which of the following is the most common type of hernia in the United States?

A

Inguinal hernia

243
Q

. Granulation tissue over the floor of the osseocartilaginous junction

A

characteristic of osteomyelitis associated with malignant otits externa.

244
Q

demonstrated by: QRS duration >120 ms; rsr′, rsR′, or rSR′ in leads V1 or V2; ‘slurred’ S wave of greater duration than R wave or greater than 40 ms in leads I and V6 in adults.

A

R BBB

245
Q

characterized by dilation of the ventricles and impaired strength of cardiac contraction, resulting in systolic dysfunction. There are multiple causes, including genetic abnormalities, alcoholism, and chemotherapy toxicity.

A

Dilated cardiomyopathy

246
Q

painless vision loss. Fundoscopic examination with reveal a cherry red fovea.

PAINLESS

A

Retinal artery occlusion

Patients with a retinal artery occlusion present with painless vision loss. Fundoscopic examination with reveal a cherry red fovea.

247
Q

Which of the following cardiac serum markers is the first to elevate following a myocardial infarction?

A

Myoglobin

Though troponin levels are more diagnostic of true infarction, myoglobin levels are the first to rise. Myoglobin usually rises within 1-3 hours after cardiac muscle injury.

248
Q

lateral ankle ulcer

A

arterial disease

249
Q

UC flare-up tx

A

Methylprednisolone

Maintenance of fluids and corticosteroids are the mainstay of treatment during acute exacerbations.

250
Q

Tissue transglutaminase IgA+ celiac disease

A

Tissue transglutaminase IgA (tTG IgA) testing is the first step in the evaluation for Celiac disease, as sensitivity and specificity is high. Total IgA should also be measured to rule out IgA deficiency, which can give a falsely negative tTG IgA. However, small bowel biopsy gives the definitive diagnosis, and is the gold standard.

251
Q

Rectal urgency and straining in an empty colon

A

“tenesmus”

252
Q

Harsh crescendo-decrescendo systolic murmur, radiates to carotids.

diamond-shaped, systolic and radiates along the aortic outflow tract.

The peaking of the murmur moves toward S2 as the valve area narrows.

–heard best at right upper sternal border

–radiates to right supraclavicular area/carotids

A

–Aortic stenosis

–left ventricular hypertrophy

–May lead to mitral regurgitation

–Elderly

–Angina, syncope, heart failure

253
Q

Which of the following valvular lesions of the heart produces a murmur that is characterized as diamond-shaped, systolic, and heard best at the right upper sternal border?

A

Aortic stenosis

254
Q

At which of the following serum concentrations of bilirubin does jaundice usually become evident clinically in an adult?

A

The classic definition of jaundice is a serum bilirubin level greater than 2.5 to 3 mg/dL in conjunction with a clinical picture of yellow skin and sclera. The discoloration typically is detected clinically once the serum bilirubin level rises above 3 mg/dL.

255
Q

acute cardiac tamponade triad

A

The classic Beck triad (low arterial blood pressure, dilated neck veins, and muffled heart sounds)

256
Q

Leads II, III, aVF

A

inferior leads

257
Q

ostium secundum is the embryologic site of occurance of which of the following congenital heart diseases

A

Atrial septal defect (ASD) is a congenital heart defect in which blood flows between the atria of the heart

258
Q

esophageal varices cause

A

Due to portal hypertension

Portal hypertension leads to enlargement of venous vasculature within the esophagus (esophageal varices).

The most common cause of portal hypertension is liver cirrhosis which can be caused from multiple diseases including hepatitis and alcohol abuse.

259
Q

A fine erythematous, papular rash with a rough texture generally located on the torso

A

This is the description of the ‘sandpaper’ rash associated with Scarlet Fever, which is seen in infection with Group A beta-hemolytic strep.

260
Q

Which of the following is the most common immediate complication following surgical repair of an abdominal aortic aneurysm?

A

Myocardial infarction

Myocardial infarction is the most commonly reported immediate complication following surgical repair of these aneurysms. It occurs in up to 10% of patients undergoing open repair.

261
Q

A 39-year-old, obese, diabetic woman comes to the clinic with bloating, nausea, lightheadedness, and early satiety. Given this patient’s history, which of the following is the most likely diagnosis?

A

Gastroparesis tx=metoclopramide.

This is a common medical finding in patients with a history of diabetes, recent surgeries (gastrectomy), and systemic sclerosis. It primarily affects the stomach’s ability to empty gastric contents, with no obstruction noted. Common symptoms include early satiety, nausea, vomiting, and abdominal pain. It can be treated with prokinetics, such as metoclopramide.

262
Q

A 2-year-old boy is brought to the office by his mother because she has noticed that the vision in his left eye has progressively deteriorated over the past month. Physical examination of the eye shows a white pupillary reflex in response to light. Fundascopic examination shows a white mass projecting from the retina. Which of the following is the most likely diagnosis?

A

Retinoblastoma

Retinoblastoma is the most common primary malignant intraocular tumor in children. It originates from the retina, and can be unilateral or bilateral. It most commonly occurs in children less than 5 years of age. Whitening of the red reflex is the most common presentation for retinoblastoma.

263
Q

A 3-year-old infant is brought to the emergency room by his parents. They state that he has been coughing with a fever for the last week, and they think he has pneumonia again. This would be his third time in the past twelve months. Physical examination shows what appears to be an undernourished child, lung sounds are coarse with peripheral wheezing, and you notice a blowing holosystolic murmur at the left sternal border. Which of the following is this patient’s most likely underlying diagnosis?

A

Ventricular septal defect

VSD is the most common congenital malformation of the heart. In children with a large VSD they may exhibit signs, as this child did, of failure to thrive (FTT), and recurrent respiratory infections. The description of the murmur is classic for VSD.

264
Q

The hallmark physical examination finding is a continuous murmur, located at the upper left sternal border, often referred to as a “machinery” murmur. The murmur often radiates down the left side of the sternum into the back, and a thrill may also be present.

A

Patent ductus arteriosus

Patent ductus arteriosus (PDA) represents 5-10% of all congenital heart lesions. Structurally, it is a persistent communication between the descending thoracic aorta, and the pulmonary artery that results from failure of normal physiologic closure of the fetal ductus. The hallmark physical examination finding is a continuous murmur, located at the upper left sternal border, often referred to as a “machinery” murmur. The murmur often radiates down the left side of the sternum into the back, and a thrill may also be present.

265
Q

Immediate referral to an ophthalmologist is indicated if an ulcer is >2 mm, if it is located directly on the visual axis, or if there is stromal melting, anterior chamber inflammation, or any scleral involvement

A

Immediate referral to an ophthalmologist is indicated if an ulcer is >2 mm, if it is located directly on the visual axis, or if there is stromal melting, anterior chamber inflammation, or any scleral involvement

266
Q

Hesselbach triangle + hernia

A

A direct inguinal hernia arises from protrusion of abdominal viscera through a weakness of the transversalis fascia of the posterior wall of the inguinal canal, medial to the inferior epigastric vessels; specifically through Hesselbach triangle. Hesselbach triangle is defined inferiorly by the inguinal ligament, laterally by the inferior epigastric arteries, and medially by the conjoined tendon.

267
Q

CA 19-9

A

pancreatic and colorectal cancers.

268
Q

A 3-year-old boy is brought to the office by his mother because of fever, rash, watery red eyes, and redness and swelling of his hands and feet for the past four days. Physical examination shows patchy generalized macular erythema, bilateral bulbar conjunctival injection, erythema, fissuring and crusting of the lips, strawberry-colored tongue, and mild erythema and edema of the palms and soles with some desquamation of the finger and toe tips. Which of the following is the most likely complication of this patient’s condition?

A

Coronary artery aneurysm

This patient has Kawasaki disease, an acute, systemic vasculitis that predominantly affects children. The condition is classically characterized by the presence of fever lasting >5 days, along with four of five of the following additional findings: bilateral conjunctival injection, oral changes such as cracked and erythematous lips and strawberry tongue, cervical lymphadenopathy, extremity changes such as erythema or palm and sole desquamation, and polymorphous rash. In the United States, it is the most prominent cause of acquired coronary artery disease in childhood. The most significant effect involves pathologic changes in the coronary arteries which may result in formation of aneurysms.

269
Q

inflammation of the margin of the eyelid.

A

Blepharitis

Blepharitis is inflammation of the margin of the eyelid.

270
Q

Anal fissures usually lie in the _________

A

posterior midline.

271
Q

Which of the following represents the recommended age to begin colonoscopy screening for a patient whose father was diagnosed with colon cancer at age 35?

A

Age 25

According to the American College of Gastroenterology, if a patient has a first degree relative who was diagnosed with colon cancer before the age of 60 years, they should begin colon cancer screening 10 years earlier than that relative’s age at time of diagnosis. Therefore, since this patient’s father was diagnosed at age 35, he/she should begin screening at age 25. Colonoscopy should be repeated every 5 years in high risk patients.

272
Q

The clinician first flexes the patient’s right hip and knee, then internally rotates the hip. A positive test is when right lower quadrant pain is elicited.

A

Obturator sign

Obturator sign is another maneuver that can be helpful in the clinical diagnosis of acute appendicitis. The clinician first flexes the patient’s right hip and knee, then internally rotates the hip. A positive test is when right lower quadrant pain is elicited.

273
Q

A 30-year-old woman is being evaluated in the office because of nasal congestion, and decreasing sense of smell over the past three months. She was diagnosed with chronic eosinophilic rhinosinusitis, nasal polyposis, and asthma six months ago. Ingestion of which of the following medications is most likely to exacerbate this patient’s asthma and nasal congestion symptoms?

A

Aspirin

Aspirin Exacerbated Respiratory Disease (AERD) [AKA: Samter triad/tetrad or aspirin sensitive asthma] is a chronic condition consisting of asthma, recurrent sinus disease, nasal polyps, and a sensitivity to aspirin and/or other non-steroidal anti-inflammatory drugs (NSAIDs). Acute dyspnea, usually accompanied by nasal symptoms - rhinorrhea and/or nasal congestion - occurs within two hours of ingestion of aspirin or NSAID. Anosmia (loss of smell) is also typical. Approximately 10% of all adults with asthma, and 40% of patients with asthma and nasal polyps, are sensitive to aspirin and/or other NSAIDs.

274
Q

Fatigue, exertional dyspnea, and orthopnea

A

Mitral valve regurgitation

Fatigue, exertional dyspnea, and orthopnea are the most prominent symptoms in chronic, severe mitral regurgitation (MR). A holosystolic murmur with radiation to the axilla is common. A S3 gallop may also be present with severe MR.

275
Q

II, III, aVF

A

inferior myocardial infarction

276
Q

28 month old infant

Physical exam shows a systolic ejection murmur with associated heave at the left-lower sternal border. You are also able to discern that there is fixed splitting of the S2 heart sound.

A

Atrial septal defect

277
Q

Which of the following physical exam findings would help the clinician distinguish between simple cholelithiasis and acute cholecystitis?

A

acute cholecystitis=murphy sign positive

278
Q

A 34-year-old man is seen in the office for consultation one day after spontaneous rupture of the tympanic membrane. While camping two days ago, he developed acute otitis media of the right ear. Yesterday, he had spontaneous relief of ear pain along with some bloody purulent discharge from the right ear canal. He has no subjective loss of hearing. Physical examination of the ear shows residual dried blood in the auditory canal, and a clot overlying a small perforation site in the pars tensa of the tympanic membrane. Which of the following is the most appropriate advice to give this patient regarding his condition?

A

. Keep the ear canal dry to allow the perforation to heal spontaneously

The auditory canal should be kept dry until the tympanic membrane perforation heals.

279
Q

Anti-double stranded DNA

A

Anti-dsDNA is highly specific for a diagnosis of Systemic Lupus erythematosus (SLE).

280
Q

A 70-year-old man presents with crampy abdominal pain,fever, and nausea and vomiting. The patient does not look well and physicial examination reveals a distended abdomen, decreased bowel sounds, and tympany with percussion. The patient has an irreducible and tender hernia and laboratory work reveals a leukocytosis and an elevated lactate. What is the most likely diagnosis?

A

Strangulated hernia

A strangulated hernia is when an incarcerated hernia progresses to the point that blood flow to the bowel is obstructed and the piece of bowel is ischemic or infarcted. Patients will present in significant pain and appear toxic. It requires immediate surgical intervention.

IF NOT TOXIC=Incarcerated Hernia

An incarcerated hernia is a non-reducible hernia, but blood flow to the incarcerated bowel is maintained.

281
Q

A patient with acute pancreatitis is most at risk for developing which of the following complications?

A

Pseudocyst

Pseudocyts usually develop late after an acute pancreatitis episode and represents a collection of pancreatic secretions that are walled off by inflammation and scar tissue. These pancreatic pseudocysts can rupture and cause bleeding and infection. In asymptomatic patients, watchful waiting is reasonable, but in symptomatic patients, pseudocysts should be drained.

282
Q

Which of the following is the most common cause of dry eye due to increased evaporative loss?

A

Meibomian gland dysfunction

Causes of dry eye is classified into two classes: 1) decreased tear production and increased evaporative loss. Both primary and secondary Sjogren’s result in decreased tear production. MGD is the most common cause of increased evaporative loss, where accessory lacrimal glands responsible for the lipid component of the tear film are dysfunctional.

283
Q

A 34-year-old man comes to the office because of increasing nasal congestion of the right side of his nose over the past four months. He states that the congestion is positional, and is usually worse when he lays on his left side. Physical examination shows a small, fleshy translucent mass on the inferior turbinate of the right nasal cavity. The remainder of the examination shows no abnormalities. Which of the following is the most appropriate initial management of this patient’s condition?

A

Intranasal application of corticosteroid spray

Topical corticosteroids are beneficial in the treatment of nasal polyps. They are effective in reducing symptoms, nasal obstruction, and polyp size. Additionally, they help in preventing polyp regrowth following surgery. Administration of oral glucocorticoids are also useful, particularly with larger polyps. Surgical excision may also be indicated with large polyps.

284
Q

You are examining a two week old infant that was brought in for his well child checkup. You note a systolic murmur at the left chest, as well as diminished pulses of the femoral arteries bilaterally.

Diminished pulses in the lower extremities and presence of a systolic murmur

A

coarctation of the aorta

285
Q

A 24-year-old man who is otherwise healthy is diagnosed with acute, uncomplicated hepatitis A infection. Which of the following is the most appropriate initial treatment for this patient’s condition?

A

Supportive therapy

286
Q

affect the nerves that run from the inner ear to the brain thus causing vertigo, ataxia, tinnitus, and hearing loss.

A

Acoustic neuroma

287
Q

A 27-year-old man and his friend come into the emergency department with severe watery, non-bloody diarrhea and low-grade fever. They state that they had dinner with about 3-4 other co-workers roughly 12 hours prior to coming in that consisted of steak, potatoes, and broccoli. They state their colleagues are having the same symptoms. Which of the following organisms is most likely to be implicated in this scenario?

A

Salmonella

Most cases of salmonella occur from eating contaminated meat or eggs. Symptoms include low-grade fever and diarrhea. The incubation period for salmonella is 6-48 hours.

288
Q

Anti-cardiolipin antibody

A

Antiphospholipid Antibody Syndrome (AAS). AAS is classically associated with a triad of antibodies: anticardiolipin, lupus anticoagulant, and an antibody that gives a false-positive laboratory test for syphillis. AAS is sometimes seen in conjunction with systemic lupus.

289
Q

Upper gastrointestinal bleeding is defined as bleeding that occurs proximal to which of the following anatomic sites?

A

Ligament of Treitz

Upper gastrointestinal bleeding is classically defined as bleeding derived from a source proximal to the ligament of Treitz. The ligament of Treitz is the suspensory muscle of the duodenum. It is a thin muscle that connects the junction between the duodenum, jejunum and duodenojejunal flexure, to the connective tissue surrounding the superior mesenteric artery and celiac artery. This suspensory muscle marks the formal division between the first and second parts of the small intestine.

290
Q

clinical characteristics of dry eyes (keratoconjunctivits sicca) and dry mouth (xerostomia)

**There is no rhematic disease involved.**

A

Primary Sjogren disease

secondary=rheumatic disease

291
Q

Dressler’s syndrome tx

A

Aspirin is most commonly recommended in patients with peri-infarction pericarditis who require anti-inflammatory therapy.

292
Q

mid-systolic murmur heard best over the (right 2nd intercostal space).

A

Aortic stenosis

293
Q

An 83-year-old man comes to the emergency department complaining of left lower quadrant abdominal pain for two days. He states that he has been feeling nauseated, and had multiple episodes of vomiting and diarrhea since yesterday. Physical examination shows a sick patient with a low grade fever, and diaphoresis. Which of the following represents the most common site affected by this patient’s condition?

A

Sigmoid colon

The sigmoid colon accounts for most cases of diverticulitis. Diverticulitis is characterized by inflammation of colonic diverticula. Patients most commonly have left sided abdominal pain, diarrhea, fever, and nausea. First-line treatment is antibiotic therapy with metronidazole and ciprofloxacin. Complications include abscess formation, obstruction, and colonic perforation.

294
Q

cobblestone-like pattern

A

Crohn disease

Skip lesions and a cobblestone pattern are classic findings on colonoscopy in Crohn disease. Segments of diseased colon are adjacent to healthy mucosa, giving a cobblestone appearance. The terminal ileum is the most common places to find ulcerative lesions and inflammation, and is the reason that many patients with this condition present with right lower quadrant pain. Perianal disease is also common in Crohn disease, including anal fistulas, fissures, and perirectal abscesses.

295
Q

x-ray shows free air under the hemidiaphragms,

A

Perforated viscus

This patient’s history of GERD and peptic ulcer disease, coupled with the physical exam findings of peritonitis (concerning for surgical abdomen), clinches the diagnosis of perforated viscus, likely secondary to peptic ulcer perforation. The x-ray shows free air under the hemidiaphragms, which is an abnormal finding and is most prominent above the liver, which is a solid organ abutting the diaphragm on the patient’s right upper abdominal quadrant.

296
Q

blood in the anterior chamber on visual inspection.

A

Hyphema

A hyphema is a collection of blood in the anterior chamber of the eye. Findings of hyphema would be evidence of blood in the anterior chamber on visual inspection.

297
Q

Which of the following anatomic sites is the most common location of an aortic aneurysm?

A

Infrarenal aorta

Aneurysms are defined as a focal dilatation of an artery, with at least a 50% increase over the vessel’s normal diameter. An abdominal aorta diameter >3 cm fits this definition. Abdominal aortic aneurysms (AAAs) usually result from degeneration of the media of the arterial wall, leading to a slow and continuous dilation of the lumen of the vessel. AAAs usually affect elderly white men, and smoking appears to be the most strongly associated risk factor. Most AAAs are asymptomatic, and many are detected as incidental findings as part of diagnostic imaging obtained for another reason. Ultrasonography is the standard imaging tool for AAA. Most AAAs (~90%) begin below the renal arteries, and end above the iliac arteries.

298
Q

A 24-year-old man comes to the urgent care clinic because of nausea, vomiting, vertigo and some hearing loss in both ears for the past one day. He states that three days prior, he had runny nose, mild sore throat, and some plugging of both ears. Which of the following is the most likely diagnosis?

A

Labyrinthitis

Labyrinthitis is an inflammation of the inner ear that results in severe vertigo, tinnitus, and possibly hearing loss, nausea and vomiting. It can occur as a single attack, a series of attacks, or a persistent condition that usually diminishes over time. Vestibular neuronitis may also be associated with nystagmus. The etiology is not always clear, but it may be associated with infections (viral or bacterial), head injury, or allergy.

299
Q

Complications of ___________include bleeding, deep structure penetration, perforation, and gastric outlet obstruction.

A

peptic ulcer disease

300
Q

Patients present with symptoms of right-sided heart failure with increased jugular venous pressure, abdominal ascites, and lower extremity edema. They may have a parasternal lift due to right ventricular enlargement.

A

Tricuspid valve regurgitation

Tricuspid valve regurgitation is most often caused by secondary hemodynamic load on the right ventricle, but may be due to a valve abnormality (e.g. infective endocarditis, rheumatic fever, carcinoid syndrome or pulmonary hypertension). Patients present with symptoms of right-sided heart failure with increased jugular venous pressure, abdominal ascites, and lower extremity edema. They may have a parasternal lift due to right ventricular enlargement. They will have a holosystolic murmur that increases with inspiration.

301
Q

skip lesions

A

Crohn disease usually presents with transmural, ulcerative lesions anywhere from the mouth to the anus that are generally non-contiguous (‘skip lesions’). Segments of diseased colon are adjacent to healthy mucosa, giving a cobblestone appearance. The terminal ileum is the most common location for these lesions. Perianal disease is also common in Crohn disease, including anal fistulas, fissures, and perirectal abscesses.

302
Q

Raynaud’s phenomenon treatment

A

Nifedipine or amlodipine

Raynaud’s phenomenon results in cyanosis of the digits caused by arterial vasoconstriction due to the sympathetic system being hyperactivated in response to changing temperatures (cold). Severe vasoconstriction may result in an infarct at that site. The treatment of choice for this condition is a dihydropyridine calcium channel blocker, such as nifedipine or amlodipine. Low dose aspirin is also recommended for patients with a history of ischemic ulcers or thombotic events.

303
Q

hich of the following is the most common cause of pulseless electrical activity?

A

Hypovolemia

304
Q

A 2-year-old girl is brought to the emergency department by her parents because they believe that she swallowed a button battery from an old camera 30 minutes ago. The girl is acting normally, and she is interactive and playful. Which of the following is the most appropriate next step in management?

A

x-ray

X-ray imaging is essential to make the diagnosis of button battery ingestion, and to confirm the exact location in the body. On the anterior-posterior x-ray study, it is useful to zoom in, and look for a double ring or halo sign to distinguish it from a coin. The lateral view x-ray can be helpful if a step-off can be noted.

305
Q

A 35-year-old woman with a history of rheumatic heart disease as a child is seen in the office for routine follow-up visit. She states that she has noticed some “jumpiness” in her pulse over the past three months, but otherwise has had no other symptoms. An electrocardiogram is shown. This patient is at highest risk for developing which of the following conditions?

A

Embolic stroke

The incidence of stroke in patients with non-valvular atrial fibrillation (AF) is between 2- and 7-fold greater than that in the general population. For patients with AF caused by valvular disease, the risk of stroke is increased 17-fold. Cardioembolic stroke is one of the main complications of AF. It occurs as a consequence of stagnant blood in the fibrillating atrium forming a thrombus from which an embolus dislodges, and enters the circulation blocking arterial blood flow and causing ischemic injury.

306
Q

mechanical obstruction of the gastric outflow tract and classically presents with epigastric abdominal pain and postprandial vomiting.

A

gastric outlet obstruction (GOO)

Malignancy

Malignancy is now the leading cause of gastric outlet obstruction (GOO) with pancreatic adenocarcinoma and distal gastic cancer being among the most common types of malignancy leading to this condition.

Gastric outlet obstruction, as the name suggests, is mechanical obstruction of the gastric outflow tract and classically presents with epigastric abdominal pain and postprandial vomiting.

307
Q

painless, gradual vision loss

associated with increase intraocular pressure, which then leads to optic nerve damage. Patients will first notice loss of peripheral vision. Risk factors include age, family history, hypertension, and diabetes.

A

Open angle glaucoma

308
Q

Fasting serum gastrin

A

Zollinger-Ellison syndrome.

309
Q

ECG by QRS duration of > 120 milliseconds, absence of Q waves in leads I, V5 and V6, broad monophasic R waves in I, V5 and V6, and ST and T wave displacement opposite to the major deflection of the QRS complex.

A

L BBB

310
Q

This combination in used in the prevention of relapse for NSAID induced ulcers in high risk patients.

A

Proton pump inhibitor, celecoxib, misoprostol

311
Q

Each of the following drugs is used in the treatment of acute decompensated heart failure EXCEPT:

A

Diltiazem is a calcium channel blocker which posses negative inotropic properties and therefore should be avoided in ADHF. Calcium channel blockers will decrease the force of muscle contraction and have been associated with poor outcomes.

312
Q

A 5-year-old girl is brought to your office by her mother who has noticed a rash on the palms of her hands and soles of her feet for the past 3 days. She also reports that her daughter has had a fever of 100.5 F (38.1 C) and significantly decreased appetite. Which of the following physical examination findings would you suspect in this patient?

A

Painful, small gray lesions on an erythematous base seen on the oral mucosa

This finding describes the aphthous ulcers commonly seen in hand, foot and mouth disease caused by Coxsackie virus A16. These patients typically have painful oral ulcers, maculopapular rash of hands and feet, fever, and refusal to eat. It is most common in kids younger than 7 years of age. It is a self-limiting disease that usually resolves in a week.